cell bio exam 3

Lakukan tugas rumah & ujian kamu dengan baik sekarang menggunakan Quizwiz!

Actin-related proteins (Arps)

(Arp1, Arp2 and Arp3) Nucleotide binding site conserved, not much sequence identity to actin, but have similar fold. Perform specialized functions as noted on slide.

patterns for the two mitotic cyclins

(Cdk2/Cyclin A, Cdk1/cyclin B) and the mitotic activity of APC/C. (Cdk 4/6/cyclin D, Cdk2/cyclin E) and the two G1/S cyclins

Basal lamina (aka basement membrane) function:

(ECM that is closely associated with the cell that produces it) Lots of names: Epithelial rug, muscle and peripheral nerve sleeve Functions: -- Physical support in a flat or tubular tissue --Barrier for cancer cellsMetastaseswhen crossed

Cell sorting experimentts (using sponges)

(cadherin = calcium-dependent adhesion) Demonstrates importance of surface densityof adhesion molecules

cytoplasmic Dynein: a minus-end directed MT motor protein

(dynein can step sideways to another protofilament; can also back up)!! Dynein motor is a dimer of two heavy chains (+ other proteins) MT-binding sites far away from AT P-binding site 15nm 6 AAA domains fused into a single polypeptide processive motor

3 kinds of functional Mts in spindle apparatus (used for assembly and function)

*kinetochore microtubules* attach to kinetochore --> two halves of spindle are mirror images *interpolar* microtubules interdigitate (overlap) also have *astral* microtubules at the spindle poles centrosomes at the minus end: polarity

a burst of Cdk1/cyclin B kinase activity in the nucleus. This proceeds to

*phosphorylate the lamins*, envelope breaks down, and cells enter mitosis.

anaphase B

- interpolar MTs slide past each other using motors (kinesis-5 aka bim c) to separate poles from each other - astral mts are pulled by dynein on plasma membrane separating the 2 poles

Tight junctions (think rivets or staples)

-- Like many lines of zip locks! Tightness of junction can be modulated. -- Define apical from basolateral membranes; highly limited permeability depending on cell type.

intermediate filaments in nerve cells Called neurofilaments

--> expand axon diameter

Spindle assembly: Three classes of spindle microtubules

1) *Kinetochore microtubules attach to kinetochores* Kinetochores: plates; assemble in the region of centromeric DNA; motors, like a kinesin called CENP-E, associated with corona. Kinetochore polarity: one on one sister chromatid faces one way, other the opposite way. When both get microtubules (late prometaphase), wind up ("congress") at metaphase plate. Kinetochores "capture" kinetochore microtubules at their plus (unstable) ends; "rescues" them from catastrophe. 2) *Interpolar microtubules*, don't get captured by kinetochores and instead grow into the other half of the spindle where they line up with counterparts coming from the other centrosome; hence their alignment is called antiparallel. Their interaction is stablilized by motors and MAPs (like PRC1 that bundles antiparallel microtubules- just FYI). 3) *Astral microtubules.* Some astral microtubules reach out and make contact with plasma membrane where the motor protein dynein "reels them in" during anaphase B

5 stages Protein destruction in cell cycle control

1) Early G1: CDK low, allows chromosomes to decondense, nuclear envelope reassembly. 2) G1/S transition: D cyclins --> Cdk4/6 pulse of activity, pass restriction point, switch to Cdk 2-cycin E--> DNA synthesis. 3) mid-S/G2: cyclin E goes, replaced by mitotic cyclins 4) Early mitosis: P --> M: active mitotic CDKs (Cdk1-cyclin B & Cdk2-cyclin A). Prometaphase, APC/C targets cyclin A 5) Late mitosis: Anaphase onset, dominated by active APC/C, both A&B cyclins targeted by APC/C, CDK activity low.

Nucleators In a cell, actin filament formation is tightly controlled and requires nucleators. Different nucleators create different arrangements of actin filaments:...

1) Formin: creates linear or unbranched actin filaments 2) Arp2/3 complex: creates branched actin filaments Formin is a dimer. Each monomer binds to G-actin and also has binding sites for Profilin (which itself is bound to ATP-actin). Formin stays at the growing barbed-end as new G-actin submits are added below the Formin dimer complex. Profilin binding to Formin helps to increase G-actin concentration near the growing barbed end. Arp2/3 complex is activated by another protein called WASP. WASP itself is activated by certain G-proteins, then binds to Arp2/3 and G-actin and initiates branch-form actin assembly.

Myosin and actin in cleavage furrow

1) Myosin forms small bipolar myosin minifilaments 2) Ca2+ influx activates myosin light chain kinase 3) Myosin motors active slide actin filaments towards pointed end (contractile ring)

Four checkpoints:

1) Restriction point in G1, looking for growth signals; 2) DNA damage checkpoint in late G1; 3) DNA damage again and proper replication, late G2; 4) chromosome attachment to spindle.

3 steps ER --> Golgi (Coat protein = COP II; GTPase= Sar1) - anterograde

1) Select cargo in ER membrane (happens at "active sites" devoid of ribosomes ---- Cargo selection in ER transitional membrane (active zone) 2) Form carrier vesicle (coat --> bud --> vesiculate--> uncoat) ----- Container formation and disassembly involving COPII and Sar1 3) Target and fuse vesicle with Golgi membrane (targeting and fusion considered later)

Neutrophil (Polys) capture and migration Inflammatory signals (cytokines) 5 steps

1) Selectins stored in vesicles move to endothelial cell surface (modulation) 2) Neutrophil *mucins*(always present) bind to the *selectins* rolling (slows them down) 3) Neutrophil *integrins* activated by chemoattractants(e.g.N-formyl peptides from bacteria) 4) Activated *integrins* bind to I-CAMs on endothelium immobilize leukocyte 5) Neutrophil moves into tissue by crawling between endothelial cells

Checkpoint #3: G2 DNA-repair checkpoint just remember 3 things about it:

1) Some are involved in detecting the damage; 2) others are kinases, like ATM, activated when damage is detected; 3) others, when phosphorylated by these kinases, are involved in keeping the cell from entering mitosis until damage repaired or, once again, activating apoptosis, via p53 and other players, if repair fails.

Two things needed to activate Cdk:

1) cyclin binding and 2) phosphorylation of T-loop. 1) Cyclin binding--> T loop gets out of the way (misoriented alpha helix rotates by 90 degrees) and gets ATP ready to donate phosphate. 2) Phosphorylation by CAK: A second Cdk/cyclin called CAK comes in and phosphorylates T loop at T160 --> stimulate enzyme activity 300x over what cyclin can do.

Cdk1/cyclin A also has a role in mitosis:

1) increase in microtubule dynamics --> "searching" for kinetochores; 2) chromosome condensation and kinetochore assembly. More later. Both cyclin A and cyclin B are targeted for destruction by APC/C once cells have initiated or are into mitosis --> can re-enter G1 at the end of cytokinesis.

kinesin movement Distinctive features of this:

1) kinesin moves along single protofilament because never lets go;processive, coordination between heads leads to long movements (100s of steps) in contrast to myosin where heads act independently; 2) large step size for a small head

Resistance to cancer is due to two factors:

1) multiple genetic changes needed in any one cell for that cell to go cancerous; 2) most of the time, if such changes occur, that cell undergoes apoptosis rather than continuing. Most cancer cells have mutant versions of one of the many players operating to maintain the gates at restriction point.

Checkpoint #1 G1 restriction point Restriction point has 2 key players manning the gate:

1) non-phosphorylated retinoblastoma protein(pRb) (will call it Rb) and 2) a family of transcription factors called E2F. To pass the gate, need to phosphorylate Rb, which then dissociates from E2F so E2F, sitting on its target genes but blocked by Rb from activating them, can now lead to expression of key genes needed to go into S, including those encoding DNA polymerase etc. and those encoding new CDKs and cyclins needed for S-phase. Rb kept from being phosphorylated during most of G1. Chromatin is also compacted via the enzyme histone deacetylase, helps Rb block transcription.

4 uses of The cytoskeleton:

1) shape cells, 2) cell division, 3) cell motility and 4) act as tracks for motor proteins (for guided movement)

3 patterns of growth regulation, as seen in skin

1) stem cells self-regenerate; 2) stems activated, divide to yield 1 stem and 1 that rapidly divides (basal layer); 3) cells stop dividing and differentiate into spinous layer. Pushed up. When reach granular layer, undergo apoptosis.

Four ways to get intracellular movements:

1) transport cargo along MT by kinesin or dynein; 2) cargoes linked to polymerizing/depolymerizing MTs; 3) transport cargo along actin by myosin (where cargo can be another actin filament --> contraction); 4) cargoes linked to polymerizing/depolymerizing actin

Functions of the cytoskeleton

1)Structural support (cell shape) 2)Organize cytoplasmic contents (organelle positioning by motor proteins) 3)Force generation a)Polymers exert pushing or pulling forces (cell motility) b)Motor proteins bound to cytoskeleton exert force (vesicle transport) c)Frequently, both work together (mitosis)

Fast axonal transport

1-2 μm/sec anterograde and retrograde) Anterograde (away from MTOC) via kinesins and retrograde(towards MTOC) via dyneins = movement on microtubules in axons.

State one reason for why kinesin-1 is a processive motor protein

1. Cooperation (reciprocal affinities) between the two heads: when one head is tightly bound, the other is weakly bound 2. At least one head is bound at every point of the ATPase cycle, so motor doesn't detach easily

Important points of kinesin ATPase cycle

1. Cooperation (reciprocal affinities) between the two heads: when one head is tightly bound, the other is weakly bound 2. At least one head is bound at every point of the ATPase cycle: --> motor efficiency high 3. One step per ATP consumed

Cell cycle runs on 2 key biochemical principles:

1. Cyclin-dependent kinases • phosphorylation/dephosphorylation • cyclin synthesis and degradation 2. Ubiquitin-dependent proteolysis • substrates: cyclins, structural proteins • ubiquitination by APC/C • degradation by proteasomes

Two key biochemical features of the cell cycle

1. Cyclin-dependent kinases (CDK) (prophase, metaphase) • cyclin synthesis and degradation • phosphorylation/dephosphorylation 2. Ubiquitin-dependent proteolysis (anaphase, telophase, cytokineses) • substrates: cyclins, structural proteins • ubiquitination by APC (anaphase promoting complex)/C (cyclosome) (or SCF) = E3 ligase • ---degradation by 26S proteasomes

2 contradictory functions of Katanin: a microtubule severing protein

1. Rapidly disassemble microtubules (eg., to change cell shape or transport) 2. Source of new microtubules (MT fragments serve as seeds to polymerize new MTs)

Elastic fibers in artery

2 billion cycles of stretching and recoil per human lifetime Elastic fibers are similar to rubber --> prominent in connective tissue of skin, arterial walls and lungs. They recoil passively after tissues are stretched.

Cadherin family

80 genes, many cell-type specific Extracellular CAD domains Single TM segment Various cytoplasmic domains; many bind catenins which mediates connection to actin cytoskeleton

Tau family of microtubule binding proteins

98% reduction in catastrophies Bundle MTs and maintain spacing between them

Kartagener's syndrome

= inherited disorder when ciliary motility impaired: immotile sperm, respiratory problems.

MAPs:

>100 microtubule-associated proteins. Influence stability and organization of microtubules.

During co-translational import of the LDL receptor at the ER, in which direction would the LDL receptor's ligand binding domain face with respect to the ER membrane? a) The ligand binding domain would face the ER lumen b) The ligand binding domain would face the cytoplasm c) The ligand binding domain would get cut off by the signal peptidase

A

Select the statements that correctly apply to intermediate filaments. a) Intermediate filaments get their name because the diameter of these filaments is intermediate compared to F-actin and microtubule diameters b) Intermediate filament subunits are generally expressed in a cell type-specific manner c) Intermediate filaments lack polarity d) Intermediate filament-like proteins are not found in bacteria e) Intermediate filaments are very strong because of their triple-helix structure f) Intermediate filaments are very strong because of their braided structure g) Intermediate filaments are very strong because of motor proteins exerting pulling forces on them

A B C F

Circle ALL statements that correctly apply to cellular membranes: a)Certain lipids are distributed asymmetrically between the two membrane leaflets b)Cholesterol and sphingolipids can form lipids rafts in the outer membrane leaflet c)GPI-anchored proteins in the plasma membrane face the cytoplasm d)All transmembrane proteins require non-ionic detergents to extract them from membranes e)The red blood cell membrane does not deform at all because of its tough spectrin-based cytoskeleton f)Some transmembrane proteins show very little diffusion within the membrane

A B D F

Which of the following statements are true of the KcsA potassium channel: a)Potassium ions are selectively transported over sodium ions b)The S5 α-helix is responsible for establishing the selectivity filter c)KcsA tetramer contains a single pore for ion transport d)This channel is voltage-gated

A C

Cadherins are distinguished by the following properties: (choose all correct statements) (4pts) a) Cadherins require Ca2+ for their interactions b) Cadherins form tight connections with specific integrins c) Cadherins are part of desmosomes in skin cells d) Cadherins are a major part of focal contacts formed by cultured cells e) Cadherins engage in homophilic interactions, unlike ICAMs which engage in heterotypic interactions f) Contact inhibition is a consequence of cadherin-based cell adhesion and signaling g) Overexpression of cadherins is associated with transition from benign to invasive malignant tumor

A C E F

Circle the statements that correctly describe motor proteins a) Myosin and kinesin motor proteins share structural similarity in their head domains b) Kinesin and dynein motor proteins share structural similarity in their head domains c) Kinesins can be either processive or non-processive d) Intermediate filament-based motor proteins typically move in a bidirectional manner e) F-actin and microtubule-based motor proteins typically move in a unidirectional manner f) Phosphate release causes the power stroke in all motor proteins g) The tail domain of motor proteins is typically the part that binds to cargo

A C E G

Select all statements below that correctly apply to emergent properties. a) The nothing buts remain unchanged during emergent properties b) Emergent properties are rarely found in non-life c) Emergent properties exist prior to interactions between the nothing buts d) The nothing buts of one emergent property may themselves be an emergent property of even smaller nothing buts (i.e., one can have multiple layers of emergent properties)

A D

Select the choices that correctly apply to lipid-based protein sorting. a) Phospholipids are synthesized in the ER while sphingolipids are synthesized in the Golgi b) Cholesterol prefers to associate with phospholipids containing unsaturated fatty acids c) Cholesterol concentration is highest in the ER because it is synthesized there d) The membrane location of a protein can be experimentally changed by changing the length of its transmembrane α-helix e) "Islands" of cholesterol and phospholipids are called "rafts"

A D

. Select the statements that correctly apply to extracellular matrix (ECM) molecules. (3pts) a) ECM molecules tend to be filamentous proteins that form an interactive network b) ECM molecules tend to be globular proteins that polymerize to form a meshwork c) Fibronectin acts as a lubricant in joints d) Fibronectin mediates matrix-matrix and cell-matrix interactions e) Entropy drives the spontaneous return of a stretched elastic fiber to a contracted state f) ATP hydrolysis drives the active return of a stretched elastic fiber to a contracted state

A D E

Circle ALL statements that correctly apply to integrins: 4 a) Integrins mediate cell-ECM and cell-cell interactions b) Integrins interact only with other integrins c) Integrins help membrane trafficking by tightly tethering vesicles to the plasma membrane d) Integrins are found to be clustered at focal contact sites e) The conformation of the integrin extracellular domain determines its affinity for the ligand f) Integrin-ligand interaction depends on Ca++ g) Some snake venoms contain integrin-binding motifs and are hence called disintegrins

A D E G

Select all statements that correctly apply to microtubules. a) EB1 is a plus-end tracking protein that can link microtubules to other cellular structures b) Intermediate filaments are more rigid than microtubules c) The critical concentration for tubulin is lower at the plus-end than the minus-end of microtubules d) Dynamic instability of microtubules occurs only with purified tubulin in a test-tube and not in cells e) Dynamic instability is a natural property of microtubules, both in the test-tube and in cells f) The GTP bound to alpha-tubulin gets hydrolyzed to GDP during microtubule polymerization

A E

Mitosis

A continuous process- the names simply indicate hallmarks • Cdk activity dominates till metaphase • APC/C activity (proteasome) dominates after metaphase

What does processive motor protein activity mean?

A processive motor is one that takes many consecutive steps before unbinding from its cytoskeletal track. [That is, binding to microtubules is followed by many stepping events (not just one step and then unbind)]

What is the mechanism by which ADP secreted by platelets bound to the basal lamina facilitates platelet aggregation?

ADP initiates signaling that activates fibrinogen receptor (fibrinogen-binding integrin) on platelets These platelets bind to each other through fibrinogen "linkers", leading to aggregation

After death, animal bodies enter rigor mortis. What stage of the myosin-II ATPase cycle is this associated with and what is the reason for stiffening of the body? ATPase cycle stage Reason for stiffening

ATPase cycle stage: Empty state (no nucleotide bound) Reason for stiffening: Muscles stuck in contracted state OR myosin heads cannot let go of the actin filament

Why does the release of acetylcholine by a motor neuron depolarize the associated muscle cell plasma membrane (aka sarcolemma)?

Acetylcholine binds to the Ach receptor (or channel) causing it to open and conduct cations (Na and K) OR Acetylcholine gates channels in the sarcolemma which conduct cations into the cell

Starting with motor neuron depolarization, order the following events in the sequence that they occur during skeletal muscle contraction. #9 has already been placed next to the last event in the series; place a #1 next to the first event, a #2 next to the second, and so on Acetylcholine secretion: Acetylcholine receptor activation: Dihydropyridine receptor activation: Muscle contraction: Ryanodine receptor activation: Sarcolemma depolarization: T-tubule depolarization: Tropomyosin positional shift: Troponin-C activation:

Acetylcholine secretion: 1 Acetylcholine receptor activation: 2 Dihydropyridine receptor activation: 5 Muscle contraction: 9 Ryanodine receptor activation: 6 Sarcolemma depolarization: 3 T-tubule depolarization: 4 Tropomyosin positional shift: 8 Troponin-C activation: 7

Cytokinesis overview

Actin and myosin based contractile ring. Contractile ring--> cleavage furrow. Bipolar myosin II minifilaments and F-actin accumulate at equator; actin attached to membrane at barbed ends. Ca transient activates myosin light-chain kinase--> ends up activating myosinII--> pulls on actin filaments (as in sarcomere)--> contracting ring that pinches the cell into two.

Focal contacts between cultured cells and ECM

Actin cables called *stress fibers* - Orange = antibody to phosphotyrosine at focal contacts - Green = phalloidin

Provide a specific example by which the actin cytoskeletal and microtubule cytoskeletal systems contribute to the following functions: Function Cell shape (shaping of a particular part of a cell is okay) Cell motility Intracellular transport (eg., transport of organelles)

Actin cytoskeleton One of the following: Formation of lamellapodia (pseudopod) by branch-form actin; formation of villi or filipodia by unbranched actin; formation of phagocytic cup by branch-form actin; formation of growth cone by branch-form actin; stress fibers due to unbranched actin; cytokinesis due to cortical actin; contraction of muscle cells (other examples also okay) Ameboid movement (in many different cell types) due to branch-form actin Transport of vesicles or organelles by myosin V (or Myosin XI) along actin Microtubule cytoskeleton One of the following: Formation of axons and dendrites; columnar epithelial cells; muscle cell morphology; formation of flagella/cilia (NOTE: anaphase B or spindle elongation is incorrect because it does NOT change cell shape, only the spindle shape changes) Movement of sperm/algae/etc. due to cilia/flagella One of the following: Transport of vesicles/organelles/chromosomes by kinesin-1 or cytoplasmic dynein; IFT by kinesin-2 or IFT dynein.

ACTIN CYTOSKELETON - Actin localization

Actin localization a) rhodamine-labeled phalloidin b) antibody Phalloidin-toxin from death cap mushroom. Binds and stabilizes actin filaments Very abundant protein - Actin can be 15% of cell protein, 60% of muscle protein. Long polymers of actin Arranged in helical ribbons around the cell just under the cell membrane Contribute to cell shape

How does clearing the spindle assembly checkpoint lead to separation of sister chromatids?

Active APC/C ubiquitinates securin (or leads to destruction of securin by the proteasome) [Since securin inhibits the separase enzyme] Loss of securin leads to active separase which degrades the cohesion complex that binds sister chromatids together

Adherens junction

Adherens junction: cadherin (+ catenin) linked to actin cytoskeleton Desmosome: "Desmo" cadherin (+ desmoplakin) linked to intermediate filaments (like keratin)

Checkpoint #2: Monitor DNA integrity

After passing through restriction point, have another hurdle: monitor DNA damage. Arrest until fixed; if not fixed, usual option is apoptosis.

Integrins

All over the place, main receptors for ECM. Lots of integrin ligands have RGD motif. Affinity for ligand can be modulated by signal transduction (activation). Promiscuity: a single integrin can bind to different kinds of ligands and a single ECM protein can bind to many integrins. Heterodimers, alpha/beta. Combinatorial code slide gives the idea - don't need to remember specific examples *Low and high-affinity states generated both by ligand binding and by signals from cytoplasm.*

Cells can respond strongly to even very small quantities of stimuli/ligands. Explain the feature of signaling pathways that enables this property

Amplification in signaling pathways allows a small stimulus to produce a substantial cellular response.

anaphase A and B

Anaphase A: Kinetochore mts shorten, daughters move to poles. Anaphase B: Poles move apart, aiding the segregation process. As noted earlier, interpolar mts slide apart via plus-end-directed kinesin- 5 motors, thereby pushing the poles away from each other ("receding goalposts"). Aster microtubules also play a role in separating poles by being pulled by dynein at plasma membrane.

Cadherin homotypic interaction

Antiparallel interaction of the first CAD domains of 2 cadherin molecules (trans-interaction) [Ca2+ ions (3 of them) keep the CAD domains more rigid]

State one difference between properties of intermediate filaments and actin filaments

Any one of the following: 1. IF subunits vary in size, while actin subunit does not 2. IFs do not have polarity, while actin filaments are polar 3. Different IFs are expressed in specific tissues, whereas actin is the same in all tissues 4. IFs have higher tensile strength than actin filaments

You discover a new protein that localizes to the plasma membrane of rod cells. However, you find that this protein lacks any transmembrane α-helices! Describe a possible mechanism for how this protein can nonetheless be plasma membrane associated

Any one of the following: Beta-sheets that span the membrane (note, this is mainly a bacterial thing, but I'll accept it); Prenylation (farnesylation); Myristylation (addition of fatty acid); GPI anchor; Binding to a transmembrane protein; Electrostatic interaction with lipids

Give one example of how a pathogen can take advantage of endocytosis to enter the cytoplasm of a host cell.

Any one of the following: Listeria: bacteria endocytosed/phagocytosed and enters endosome. There it produces proteins (lysins) that break the endosome membrane allowing the bacteria to enter the cytoplasm. Diphtheria: bacteria endocytosed and in the endosome, the T-domain of toxin permeabilizes the endosome membrane to allow the toxic A domain to enter the cytoplasm. Flu virus: flu virus endocytosed. In the endosome, the hemagglutinin protein changes conformation and hooks into the endosome membrane, then changes conformation again as pH gets lower to cause endosome and viral membrane fusion, virus exits into cytoplasm.

During phagocytosis, what is the mechanism that localizes actin polymerization to the phagocytic cup (i.e., the site where a bacterium is attached to the macrophage plasma membrane)?

Any one of the following: i)Localized production of PIP3 at the phagocytic cup ii)Localized activation of G-proteins that stimulate actin polymerization at the phagocytic cup

Actin and actin-related proteins (ARPs)

Arps have different functions: Arp1 part of dynactin complex (dynein cofactor) Arp2/3 complex initiates actin filaments

negatively charged, hydrophilic "acidic amino acid" DE

Aspartic Acid (Asp) glutamic acid (Glu)

Axoneme

Axonemal dynein on A-tubule (attache) walks on adjacent B-tubule --> produces sliding --> driving motility of the cilia or flagella Sliding of outer doublets somehow produce axonemal bending doublets are sliding with respect to each other. A structure found in eukaryotic cilia and flagella and responsible for their motion; composed of two central microtubules surrounded by nine doublet microtubules (9 + 2 arrangement).

Aspects of cilia structure to remember:

Axoneme. Outer doublets A and B microtubules, central pair (9+2), >200 other proteins including outer and inner axonemal dynein arms with ATP-sensitive stalks. Dyneins walk towards minus end, pushing adjacent tubule upwards. Somehow this sliding is converted to bending and bend propagation but not understood how.

Cilia, mussel gill epithelia

Axoneme: What 's left after membrane removed with nonionic detergent MTOC=Basal body motile flagella and cilia mostly composed of microtubules (mtoc)

Glycosaminoglycans (GAGs) are: (choose all correct statements) (3pts) a) Made only by fibroblasts b) Attached covalently to proteins called proteoglycans c) Are a major part of loose connective tissue like the mesentery (in our gut) d) Are very hydrophobic and act as water sealants in bone e) Are very hydrophilic and act as lubricants in joints f) Are the primary reason why new-born baby cheeks are so springy

B C E

Circle all statements that correctly apply to the microtubule-associated protein tau. a) The normal function of tau is to increase microtubule depolymerization b) Tau is found primarily in the axon of nerve cells c) Formation of paired helical filaments and aggregations of tau is a hallmark of Alzheimer's disease d) Hyper-phosphorylation of tau causes to it hyper-stabilize microtubules e) Tau bound to microtubules typically inhibits kinesin-1 motility f) Tau stabilizes microtubules g) Tau mediates linkage of vesicles to microtubules during axonal transport

B C E F

. Select the statements that correctly apply to nuclear envelope breakdown. (4pts) a) NEB occurs early in prophase during DNA condensation b) Phosphorylation of lamins induces nuclear envelope breakdown c) NEB is one of the hallmarks of prometaphase d) Phosphorylation of histone deacetylase induces nuclear envelope breakdown e) NEB is essential to pass the restriction point f) NEB is essential to allow spindle microtubules access to kinetochores g) Premature degradation of cyclinB during prophase would block NEB

B C F G

Select the statements that correctly describe the barbed and pointed ends of F-actin. (3pts) a) The critical concentration for G-actin is about 5-fold higher at the barbed end, which causes the barbed end to grow faster than the pointed end b) The critical concentration for G-actin is about 5-fold lower at the barbed end, which causes the barbed end to grow faster than the pointed end c) Formins and Arp2/3 complexes are both found at the base (pointed ends) of nucleated actin filaments d) Formins and Arp2/3 complexes are respectively found at the barbed and pointed ends of nucleated actin filaments e) The barbed ends are typically attached to the plasma membrane f) The pointed ends are typically attached to the plasma membrane, allowing the barbed ends to grow freely in the cytoplasm

B D E

Select all statements that correctly apply to the ECM and cell adhesion. a) Contact inhibition is typically increased in cancer cells due to greater expression of cadherins b) Calcium ions are required for cadherin function c) The basal lamina is a strong structure that can never be broken down by cells d) Integrins can only bind to ICAMs to mediate cell-cell adhesion e) Integrins can be activated by the binding of extracellular ligands and by intracellular signaling f) Addition of Ca++ activates ICAMs to promotes adhesion between sponge cells

B E

at anaphase B, kinesin-5 dominates in the *interpolar* microtubules, pushing poles apart

B - poles moving further apart (achieved by changing the forces of the kinesins to more outward dominating forces) - kinesin 5 slide Mts apart by moving toward plus end Astral microtubules are pulled by dynein on plasma membrane.

Why do we think that all of life on Earth likely originated from a single common ancestor?

Because all organisms share commonalities like DNA genome, triplet codon, ribosomes, core metabolic pathways, etc. [mention of any one of these commonalities is fine]

Platelet cells always have collagen-binding integrins in their plasma membrane. Then, why don't platelets routinely stick to the sides of blood vessels (i.e., in a healthy blood vessel)?

Because collagen is not exposed in a healthy vessel (or the endothelial cell layer is intact in healthy blood vessels)

Why do mutations in genes encoding keratin proteins commonly lead to blistering diseases

Because keratin is important for mechanical strength of skin OR because keratin connects/supports desmosomes

Explain why mutations in the intermediate filament keratin can lead to blistering skin disorder.

Because keratin mechanically supports desmosomes between skin cells OR Because attachment of skin cells (or skin epithelia) depend on desmosomes that are supported/strengthened by keratin filaments.

The Na+/K+ pump itself doesn't transport glucose in the intestinal epithelium. So why is it essential for glucose uptake?

Because the Na gradient is needed for the Na/glucose symporter to work

Give one scientific reason why we think that eukaryotes, bacteria and archaea arose from a common ancestor.

Because they all share certain core features such as DNA genome, triplet code, ribosomes, central metabolism, etc. (mention of any one of these features is fine)

Families of actin-binding proteins: Capping

Bind to either end and prevent polymerization/depolymerization

Kinesin-1 on microtubule

Binding site maps to β-tubulin associate of mtoc track Kinesins decorate each protofilament --> Walk on a single protofilament(no side steps) Kinesins walk from one beta to the next beta subunit --> *large step* MT + kinesin head + AMPPNP (non-hydrolyzableform of ATP) (-) end: alpha end (+) end: beta end

Tau and MAP2

Both in tau family. Stabilizing MAPs: bind along length and generate MT bundles (i.e., coaligned MTs). Spacing between MTs determined by length of the projection domains. MTs with tau grow faster, shorten slower, 98% reduction in catastrophes. Tau mainly in axons; MAP2 mainly in dendrites. (MAP4 is found in other cell types)

The activity of cyclin-dependent kinases (Cdks) is tightly regulated by cells. Give one example of how: a) The 26S proteasome system can influence Cdk activity b) Phosphorylation can influence Cdk activity

By degrading cyclins (degrading CAK, Wee1, Cdc25, p27 and INK4 is also acceptable even though we did not talk about this in lecture) b) Phosphorylation can influence Cdk activity One of the following - CAK phosphorylates Cdk to increase its activity; - Wee1 phosphorylates Cdk to decrease its activity; - Cdc25 dephosphorylates Cdk to increase its activity

The figure below shows an experiment in which a small fluorescent molecule (fluorescein) was injected into a single epithelial cell (labeled "3" in the picture). When this epithelium was imaged after some time, it was observed that fluorescein spread to adjacent cells on both sides. a) How is fluorescein able to spread to the cytoplasm of adjacent cells? b) If you did a similar experiment using GFP (a protein of about 25 kDa), would you expect a similar result (i.e., GFP spread to adjacent cells)? Explain your answer.

By passing through Gap junctions that exist between these cells. b) If you did a similar experiment using GFP (a protein of about 25 kDa), would you expect a similar result (i.e., GFP spread to adjacent cells)? Explain your answer. No GFP would not spread to other cells—1pt Because it's too large to pass through the gap junctions—1pt

The importance of an autocatalytic cycle in our discussion of autocell formation was: (select all correctstatements). a) An autocatalytic cycle is self-sustaining without further input from the environment b) Without an autocatalytic cycle, container or capsule formation would have been impossible c) An autocatalytic cycle produces the catalysts needed to keep the cycle going d) An autocatalytic cycle produces the substrates needed to keep the cycle going

C

What do you expect to observe if you did an in vitro motility assay with kinesin-1 bound to latex beads using a non-hydrolysable form of ATP (e.g., ATPγS)? ( a) The beads will move towards the microtubule minus-end b) The beads will not bind to microtubules c) The beads will bind but not move along microtubules d) The beads will bind and move along microtubules

C

Circle ALL statements that correctly apply to the cytoskeleton: 2 a) The cytoskeleton needs to be static in order to maintain cell shape b) In a cell, actin and microtubules assemble spontaneously depending on the subunit critical concentration c) The GTP-cap model proposes that microtubules will grow as long as their plus-end consists of GTP-tubulin d) Cytoskeletal proteins are found in eukaryotes, bacteria and archaea e) The pointed end of F-actin is thus named because it grows faster than the barbed end

C D

Select all statements that correctly describe membrane trafficking in eukaryotes. a) GTP-bound Arf1 is found predominantly in fully formed vesicles b) Clathrin coat protein mediates anterograde trafficking from ER to the Golgi c) Tethers mediate the initial recognition between vesicle and acceptor membrane d) Synaptotagmin acts as a Ca++ sensitive brake to regulate secretion e) BAR domain proteins mediate the initial recognition between vesicle and acceptor membrane

C D

Select all statements that correctly apply to the LDL receptor. a) LDL receptor endocytosis requires interaction of v-SNARE and t-SNARE proteins b) LDL receptors regulate the synthesis of cholesterol c) LDL receptors unbind LDL particles in early endosomes and get recycled back to the plasma membrane d) Inhibition of V-type proton pumps would inhibit unbinding of LDL and LDL receptors e) Dynamin is a G-protein that is required for endocytosis of LDL-bound LDL receptors f) Arf1 is a G-protein that is required for endocytosis of LDL-bound LDL receptors

C D E

Select the statements that correctly describe actin. (3pts) a) The critical concentration for G-actin is about 5-fold higher at the barbed end, which causes the barbed end to grow faster than the pointed end b) Formins and Arp2/3 complexes are both found at the pointed ends of nucleated actin filaments c) Organization of the actin cytoskeleton can be regulated by a variety of Rho-family GTPases d) The critical concentration for G-actin is about 5-fold lower at the barbed end, which causes the barbed end to grow faster than the pointed end e) Barbed ends are typically directed towards the plasma membrane f) In cells, profilin binds to G-actin subunits and keeps them in the ADP-bound state

C D E

Circle ALL statements that correctly apply to the cytoskeleton: 2 a) Cytoskeletal proteins are found only in eukaryotes b) The cytoskeleton is a static structure that serves as a structural framework of cells c) Actin severing proteins like cofilin preferentially bind to and cut ADP-actin filaments d) In a cell, actin and microtubules assemble spontaneously depending on the subunit critical concentration e) The GTP-cap model proposes that microtubules will grow as long as their plus-end consists of GTP-tubulin

C E

Select the statements that correctly apply to tubulin. (3pts) a) Tubulin-like proteins are found only in eukaryotes b) Only the β-tubulin subunit binds to GTP c) In animal cells, γ-tubulin is found predominantly at the centrosome d) All tubulin isoforms are capable of polymerizing to form microtubules e) Only the αβ-tubulin isoforms are capable of polymerizing to form microtubules f) Both α-tubulin and β-tubulin subunits bind to GTP g) α-tubulin faces the plus-end of microtubules

C E

Select the statements that correctly apply to dynamic instability of microtubules. (4pts) a) Dynamic instability is only observed in vitro (in a test tube) and not in cells b) The GTP cap refers to a GEF (guanine-nucleotide exchange factor) that keeps tubulin in a GTP state c) The GTP cap refers to the GTP state of β-tubulin at the plus-end of a microtubule d) The GTP cap refers to the GTP state of β-tubulin at the minus-end of a microtubule e) Loss of the GTP cap is thought to cause microtubule depolymerization due to the curved structure of GDP-tubulin f) Dynamic instability is thought to facilitate capture of microtubules at kinetochores g) Dynamic instability hinders capture of microtubules at kinetochores because the microtubules do not stay put h) Dynamic instability can be regulated by microtubule-associated proteins i) Dynamic instability cannot be regulated since it is an inherent property of microtubules

C E F H

Adherens junction and desmosome

Cadherin homophilic interactions Adherens junction - E-cadherins - β-catenins -->link to Actin Desmosome - "Desmo" cadherins - Catenins+ desmoplakin --> link to Intermediate filaments(keratin in skin)

Briefly describe how activation of Sar1 GTPase leads to sorting of cargo destined for the Golgi and generation of membrane curvature. Cargo sorting Membrane curvature

Cargo sorting : Sar1 effector (sec23-24 dimer) binds to cargo (and thus clusters them) Membrane curvature : Either, i) binding of Sar1 effector (sec23-24 dimer) induces membrane curvature, or ii) binding of COPII coat proteins induce membrane curvature

Activation of Cdk

Cdk alone only partially completes the folding process; mouth of catalytic pocket blocked by T loop, and another alpha helix is misoriented so ATP, although it can bind, can't transfer P (don't need to remember these details).

The G2 Phase and Control of Entry into Mitosis

Cdk1 is present at same level throughout cell cycle. One of its jobs, as indicated earlier, is to phosphorylate lamins--> nuclear envelope breakdown. It doesn't do this, however, until it is activated, so entry into mitosis is in fact controlled by regulating these activation systems. The first is that it is associated cyclin (cyclin B1; we can just call it cyclin B) gets synthesized during S and the two associate, getting Cdk activity to basal levels.This is quickly followed by 2 events: 1) activation rendered by CAK phosphorylating the T-loop atT161 AND 2) inactivation rendered by Wee1 (and a second kinase) phosphorylating (don't need to know the precise amino acids that get phosphorylated, just know that phosphorylation by CAK and Wee1 is at different positions). Cdk1 sits in this state until late G2, when the Cdc25 phosphatase removes both inhibitory phosphates.

How does Cdk1 activation contribute to nuclear envelope breakdown?

Cdk1 phosphorylates lamins (lamins A, B and C) which leads to breakdown of the nuclear cytoskeleton.

Control of entry into mitosis: Regulation of Cdk1 by cyclin B & phosphorylation/dephosphorylation

Cdk1 present througout the cell cycle, but inactive. --- Cyclin B gets synthesized in S phase, then CAK activates. But Wee1 quickly inactivates again....Cdk1 sits in this state until late G2 when Cdc25 dephosphorylates Cdk1, reactivating it.

Positive regulation of Cdk2 by cyclin A and phosphorylation by CAK

Cdk2 kinase: not active, T loop blocks access to catalytic site; minial, low level activity Cdk2 - cyclin A: when appropriate cyclin come, induces conformational change and t loop swings out of the way, exposing the catalytic site and will active the site; basal activity Active Cdk2 - cyclin A: CAK will phosphorylate *cyclin bound* cdk and is maximally active

Regulation of Cdk2/cyclin A activation

Cdk2 kinase: not active, T loop blocks access to catalytic site; minimal, low level activity Cdk2 - cyclin A: when appropriate cyclin come, induces conformational change and t loop swings out of the way, exposing the catalytic site and will active the site; basal activity Active Cdk2 - cyclin A: CAK will phosphorylate *cyclin bound* cdk and is maximally active *Wee1 kinase*, will phosphorylate other sites and *inactivates Cdk2* *Cdc25 phosphatase*- *reactivates*, revert cdk back to maximally active state

How do centrosomal versus noncentrosomal microtubule arrays differ in terms of their spatial organization

Centrosomal arrays are radial, while noncentrosomal arrays are linear OR Centrosomal microtubules are anchored/stuck to the centrosome while noncentrosomal MTs are not.

Mitosis Centrosomes and Centrioles

Centrosome a large region encircling the centrioles; in metaphase there are 2 centrosomes, each containing a centriole pair.

Anaphase: steps

Chromatids separate Happens as follows : 1) Active APC/Ccdc20 (which also ubiquitinating cyclins) tags securin with Ub and it's destroyed. 2) Securin was binding to and inhibiting a protease called separase. 3) The now-active separase can cleave a component (don't need to remember its name) of the cohesion complex, which has been holding the sister chromatids together since S phase. 4) The chromatids can now separate.

Onset of prophase

Chromosomes condense. Centrosomes separate. The centrosomes are organizing more microtubules (> MTOC activity, > γ-tubulin). Moreover, these are far less stable than interphase microtubules, with catastrophes frequent and dramatic such that the microtubules often shorten all the way back to the ring complexes before growing out again. As a result, they tend to be short (short half-life).

Tight junction Claudin or Occludin

Claudin or Occludin: transmembrane protein. Interact closely from two cells to form a tight seal. Reticulate organization. Results in apical and basolateral membrane domains.

Explain the difference between co-translational and post-translational protein targeting and give one example of an organelle that uses this mode of protein targeting. Compartment explanation

Co-translational protein targeting Compartment: ER Explanation: Co-translational protein targeting is when protein translation (synthesis by ribosome) is coupled to protein import into the organelle Post-translational protein targeting Compartment: Mitochondria or chloroplast or peroxisome Explanation: Post-translational protein targeting is when the protein is fully translated and the resulting polypeptide is subsequently imported into the organelle

Actin filament severing by cofilin

Cofilin binding locally reduces twist of helical actin strand --> Making this region more flexible. Differences in mechanical strength between rigid and flexible regions lead to shearing of filament. Severs ADP-actin filaments. Remains bound to an ADP-actin monomer

F-actin severing proteins

Cofilin likes ADP-actin and hence targets old filaments, pulls out an ADP-G-actin thus severing the filament.

Compare and contrast collagen IV and fibronectin Name the cell that produces them: Their typical biological function is:

Collagen IV --Fibroblast --Form (part of) basal lamina OR form sheet surrounding cells OR mechanically support epithelial cell layer (or other tissues) Fibronectin --Fibroblasts (and other cells like chondrocytes & myoblasts) --Mediate cell-ECM (or ECM-ECM) connections OR allow cell adhesion to ECM

Gap junction EM

Connexin: 20 genes, tissue-specific expression Connexon= 6 connexins from each cell (so, 12 per "channel") --- Six connexins oligomerize to form hemichannels called "connexons," which then align in the extracellular space to complete the formation of gap junction Can open and close (Ca2+ close connexons)

Gap junction Connexins

Connexins—Again bind to each other from adjacent cells to form a connexon "channel". Allow small solutes like sugars, amino acids, second messengers to pass through gap.

Beta-catenin links cadherins to actin filaments Contact inhibition:

Contact inhibition: Epithelial cells contact each other --> adherens junctions (cadherin) --> Signals generated that inhibit the proliferation and migration of cells. Loss of cadherin --> transition from benign to invasive malignant tumor (expression of E-cadherin in cultured cells can correct this problem)

State one function of the core particle and cap structure in the 26S proteasome.

Core particle digests/cuts substrate proteins OR the core particle is the protease unit— Cap unfolds substrate proteins (through ATP hydrolysis)

A leucine zipper is an example of a coiled-coil structure. What type of interaction is mediating the formation of this coiled-coil? (select one) a) Electrostatic bond b) Disulfide bond c) Hydrogen bond d) Hydrophobic effect Explain the reasoning for your answer:

D Leucines are hydrophobic (or non-polar) amino acids, hence they interact through the hydrophobic effect.

General features of DNA damage checkpoints

Damage --> Sensors--> Transducers-->Effectors --> Response

Explain how damage to blood vessels gets platelets to bind to the site of injury.

Damage to blood vessels exposes the basal lamina— Platelets bind to the basal lamina through integrins that bind to collagen IV (or GP1B binds to von-Willebrand factor

Actomyosin ATPase cycle

Different from the GTPase cycles we're used to in that Pi release is the rate limiting step rather than getting rid of the ADP, and it is this step that actin speeds up 200x (analogous to a GEF speeding up the getting rid of GDP). M or M-ADP bind tight (rigor), M-ATP or ADP+P bind loose, alternation allows to release and re-bind. Force produced when going from AMDP AMD (i.e., Pi release step). ADP released, new ATP binds head releases; when ATP hydrolyzed to ADP+P lever arm cocked, ready for any cycle. ADP dissociates (when there is no ATP the head is bound tightly to the actin filament) -> ATP binding, head dissociates -> ATP hydrolysis (lever arm moves into cocked position) -> rapid equilibrium, free and bound (when bound here the bond is less strong than in phase with no ATP) -> phosphate dissociates, light chain domain rotates -> repeat 3 states? 1) no actin - just myosin 2) myosin & actin: tightly bound, strong --> AM*T --> AM*DP: both weak *loss of P is rate limiting* - presence of actin greatly enhance rate of phosphate release (power stroke) Myosin moving towards barbed end [down], pointed end of actin pushed up. Also, each head acts independently of the other in myosin-II weak, weak, strong (power stroke) strong, weak

Assembly of intermediate filaments from subunits

Dimers associate, head-to-tail, to form a strand, in conjunction with a second strand, staggered, that is oriented in the opposite direction --> no polarity - both ends are equivalent, unlike actin and microtubules. These "tetramer" strands then go on to make the mature IFs, 10 nm diameter. So, IFs grow both by adding to ends and to sides (actin and microtubules: end-growth only). Not known how nucleated.

Microtubule organizing centers (MTOC):

Discrete structures that initiate MT growth 1) centrosome (contain gamma tubulin; more in mitosis section) 2) basal body

How elastin works

Driven by entropy! Heart beat --> pressurized blood stretches the arteries. •Random coil regions of elastin get stretched --> more order. •Later --> reverts spontaneously to contracted state (more disordered). Made only through adolescence; loss --> wrinkles Marfans yndrome: mutations infibrillin; aortic enlargement (aneurysm) --> rupture

Cell-cell adhesion Tissues depend on adhesion of cells to cells and cells to ECM: How does this happen?

During development, genetic programs specify cell-cell and cell-ECM interactions to shape tissues and organs.

dyenein structure

Dynein consists of 2 heavy chains, multiple intermediate and light chains—big multi-protein complex! Also, generally requires association with Dynactin—which is also a large multi-protein complex!

Motors move on microtubules covered with MAPs In a cell, MTs are decorated by nonmotile MAPs. Do these "static" obstacles impede motors?

Dynein takes side/back steps when it encounters Tau Tau causes kinesin-1 to detach or pause Non-motile MAPs like tau can significantly affect the motility of motor proteins like kinesin. Dynein motility less affected by tau

What mechanical property distinguishes elastic fibers from collagen fibers?

Elastic fibers are stretchable while collagen fibers are inextensible/strong/tough.

State an emergent property for the following structures. Structure: An αβ-tubulin dimer Protein kinase A (PKA) and its regulatory subunit RII Sarcomere

Emergent property Polarity; GTP binding Inhibition of PKA Bipolarity or mirror symmetry; Contraction

State an emergent property for the following nothing buts. Nothing buts - The amino-acid, F - Formin, profilin and G-actin - V-type H+-pump in the plasma membrane

Emergent property (any ONE listed below) Hydrophobic; ring structure; apolar (linear) Actin filament Proton gradient

Cadherins

Especially important in junctions called desmosomes and adherens junctions (later). CAD domains. Form anti-parallel homotypic interactions with adjacent cell's CAMs. Calcium needed to keep the CAM domains in place. Cytoplasmic domains often associate with *beta-catenin*, which enables *binding to actin filaments* and signaling Cells with matching cadherins bind together and exclude those that don't; major importance in epithelial tissue formation.

Name a suitable experimental technique to answer the following questions Question Is the connexin45 gene expressed in cardiomyocytes? Does EB1 localize to growing microtubule plus-ends during mitosis? Does tropomodulin bind directly to tropomyosin? Does spectrin have a role in cell motility?

Experimental technique (any ONE listed below) Northern blot; RT-PCR; RNA-Seq; [also Reporter gene (i.e, connexin45 promoter followed by GUS or GFP etc.) is fine] GFP technology (i.e., image EB1-GFP in living cells). NOTE: Immunofluorescence and electron microscopy are wrong because microtubules are not growing in these cases! Yeast 2-hybrid; immunoprecipitation; pull-down; affinity chromatography; mass spectrometry RNAi; mutagenesis

Name one protein that performs the functions listed below for F actin and Microtubule Nucleating protein End-binding protein

F-Actin - Formin OR Arp2/3 complex (VASP also OK although we did not talk about it in class) - Capping protein/CapZ/Gelsolin/Tropomyosin/Tropomodulin/Arp2/3 complex Microtubule - γ-tubulin (ring complex); GCP proteins OK too although we didn't talk about them - EB1/EB3/CLIP170/γ-tubulin complex/CAMSAPs

Fibrillar and sheet-forming collagens - Fibrillar collagen (type I): - Sheet-forming collagen (type IV):

Fibrillar collagen (type I): made by fibroblasts; provide tensile strength to tendons, ligaments, etc. By weight, fibrous collagen is as strong as steel (tensile strength). Collagen I fibers are inextensible because of the elongated triple helix structure (cannot be stretched any further) Sheet-forming collagen (type IV): sheets surround epithelia. Form basal lamina in muscle and nerve cells.

Overview of Cytoskeleton and Cellular Motility

Filamentous network inside cells. Very dynamic. Three kinds: actin, microtubule and intermediate filaments. Have different size and mechanical properties (emergent) Determine cell shape, cytokinesis, symmetry, chromosome movements. Much diversity on general themes.

Filopodium pseudopod or lamellum

Filopodium = narrow extension (forming involved) pseudopod or lamellum = broad extension (Arp2/3 involved).

Major landmark of G1 = Restriction point

First checkpoint - check for nutrients and mitogens are continuously present, are conditions good for division Cell checks for: Nutrients & growth factors (will just grow in this phase until ready) once it passed this checkpoint, cell is committed to divide

Briefly explain how H+ flow through the F0 subunit of the F-type ATPase leads to conformational changes in the β-subunits of the F1 subunit (you do NOT need to explain the different conformational states of the β-subunits and how they relate to ATP synthesis!).

Flow of H+ through the F0 subunit causes the F0 subunit to rotate which in turn causes the gamma-subunit/stalk/camshaft to rotate (the key point) which induces conformational changes in the F1 subunit.

Focal contacts between cultured fibroblast and ECM (fibronectin)

Focal contacts: specialized sites with integrin clusters and linkage to cytoskeleton and signaling Orange= antibody to phosphotyrosine-- signaling! Green= phalloidin --> Think of focal contact sites as signaling hubs informing cells of the ECM environment

Filopodia and ruffles on cultured cells

Formin (red) concentrates at tips of filopodia --> finger-like extensions Arp2/3 concentrates at lamellum --> broad extensions

Formation of trimer difficult for actin elongation/polymerization in vitro Problem solved in 2 ways: Formin and Arp2/3

Formin- Binds two actin monomers and locally increases monomer concentration by binding profilin-actin. Makes trimer formation more favorable. Facilitates linear filament nucleation and elongation. Arp2/3 complex - Along with WASP-actin, forms a pseudotrimer to facilitate branched filament nucleation. Bundled actin arrays require linear filaments, while lamelipodia require a branched actin network

Use the data below to explain the molecular mechanism for the spindle assembly checkpoint

Free kinetochores bind to Mad1 Kinetochore-bound Mad1 binds to Mad2 and gets Mad2 to bind Cdc20 This prevents Cdc20 from binding to APC/C, which leads to arrest in metaphase

G1 and Regulation of Proliferation - G0

G0 cells no longer cycling (can last short or long time, as with neurons). Cells are not dormant or dead, they are doing their usual thing. Go into G0 for at least 2 reasons: 1) Signals that actively suppress going into G1 (example, TGF-beta); 2) Not enough growth factors (this may also stimulate apoptosis).

Cell cycle phases

G1 - 1st gap phase (between m and s) S- first stage discovered ; DNA replication stage (s for dna synthesis) G2- second gap phase (between s and m) M - second stage discovered (m for mitosis) v important/critical feature of cell cycle are the checkpoints- makes sure certain criteria is met before it can advance to next step of the cell cycle - g1 checkpoint: restriction point - make sure condition are suitable for cell division and cell size is right -2 checkpoint: check for DNA damage -g2 checkpoint: check for damaged or unduplicated DNA - m checkpoint: check that chromosomes are alligned correctly during metaphase

The basal lamina also contains a protein called von Willebrand factor; platelets bind to it via

GP1B

Elongation of pure microtubules in vitro:

GTP-tubulin works way better than GDP-tubulin.Growth rate concentration dependent: More GTP-tubulin --> faster growth (from both ends).Plus end elongates faster than minus at given concentration.

The junction that is permeable to small molecules between cells is called:

Gap (junctions

Motor proteins result in different kinds of intracellular movement depending on whether the motor or cytoskeletal filament is anchored. Give an example of an intracellular movement in which the motor protein is itself anchored; also name the motor involved: Give an example of an intracellular movement in which the cytoskeletal filament is anchored; also name the motor involved

Give an example of an intracellular movement in which the motor protein is itself anchored; also name the motor involved: Possible answers are : Myosin-II = muscle (or sarcomere) contraction Axonemal dynein = flagella/cilia motility OR sliding of axonemal microtubules Give an example of an intracellular movement in which the cytoskeletal filament is anchored; also name the motor involved: Myosin-V = vesicle/organelle transport Kinesin-1 = vesicle/organelle transport OR axonal transport Cytoplasmic dynein = vesicle/organelle transport OR axonal transport IFT dynein = (retrograde) IFT Kinesin-II = (anterograde) IFT

Special amino acid and uncharged GCP

Glycine (Gly); nonpolar Cysteine (Cys); POLAR Proline (Pro); non polar

G0 phase

Going into G0 is a normal part of differentiation, no longer able to divide. Go into G0 if suppressed (e.g. by TGF-β inhibits cell division) or if not enough growth factor signaling. Many proteins and activities regulate whether to go in and come out. G0 cells are doing everything else, just not cycling; not dead, metabolically active, just not dividing A nondividing state occupied by cells that have left/ exited the cell cycle, sometimes reversibly.

Protein destruction in cell cycle control Big picture

Going into mitosis, Cdk1/cyclin B phosphorylates lamins, gets Nuclear envelope to disassemble; Cdk2/cyclin A thought to be involved in setting up spindle and in chromosome condensation. Once in mitosis, want to kill both these activities. Way that happens is that the cyclins are targeted for destruction, and once the Cdks can no longer continue to phosphorylate, phosphatases move in and cell reverses mitotic state. Securin, a protein involved in maintaining sister-chromatid attachment, is also ubiquitinated and destroyed.

What causes the plasma membrane to extend forward during amoeboid cell motility?

Growing barbed-ends of branch-form actin filaments (or Arp2/3 nucleated actin filaments) pushing on the membrane

dynein ATP binding site

Has six ATP binding sites (AAA domains) but only one is important for motility (AAA1). Long narrow stalk projects from AAA ring that has MT binding site on its tip. The linker domain that drapes across the AAA ring is the lever arm that moves during the ATPase cycle. The ATP-dependent conformational changes are thought to be communicated to the MT-binding site (that is located far away from AAA1) by dynein-specific structural motifs like the C-term, strut and coiled-coil stalk (that has the MT-binding domain at its tip).

skeletal Myosin head

Heads: N-term of heavy chain has catalytic domain with actin-binding site and ATP-binding site. In some myosins, this is followed by a long coiled-coiled and then tail domain. 2 heads (dimer) Essential light chain Regulatory light chain protease cleavage site

Cadherins in inter-cellular junctions linking cells together

Homophilic interactions Requires Ca2+(break using EGTA- a Ca2+ chelator)

Flagellar growth and intraflagellar transport (IFT)

How do flagella grow - cross regular Chlamy (green algae) with regenerating flagella with opposite mating type expressing GFP - labeled tubulin - fused cell now has GFP tubulin, which will get incorporated in the growing flagella - if you shear Chlamy in a blender, typically only one flagella breaks. How they know that flagella have reached same length is unknown anterograde and retrograde IFT powered by Kinesin-2 family and by the *IFT- dynein* motor, respectively *flagella grow from their tips (or plus-ends) [and not from the ends facing the basal body]* c) What is the name of the microtubule organizing center (MTOC) for flagella? ___basal body (not axoneme!) d) What is the function of intraflagellar transport (IFT) in these cells? *Transport flagella building blocks* [i.e., tubulin, dynein, radial spoke parts, etc] *to the growing flagella tip* and recycle stuff back into the cell [usually to be degraded].

3 steps Golgi --> ER (Coat protein = Cop I; GTPase= Arf1) - retrograde

I) Select cargo in Golgi membrane (eg., ER-resident proteins) 2) Form carrier vesicle (coat--> bud--> vesiculate/uncoat) 3) Target and fuse with ER membrane

Adhesion molecules

ICAMs and LFA integrins in lymphocyte/Ag-presenting-cell interactions

Explain in general terms (i.e., specific protein names are not required) how G-proteins contribute to the formation of a vesicle starting from a flat membrane.

In a flat (donor) membrane, G-proteins are activated (i.e., in GTP state) by GEFs and they then bind to (or insert in) the membrane The active G-proteins recruit coat proteins (or effectors) that bend the membrane to form a bud In the curved bud membrane, GAPs get activated, leading to inactivation of G-protein (GDP state), and uncoating to form a vesicle

Profilin as a gatekeeper for actin distribution between formin & Arp2/3

In cells, only a fraction of the actin monomer pool is bound by profilin. Profilin-actin subunits are used preferentially by formins(and Ena/VASP) to build unbranched actin structures, while free actin subunits are used more favorably by the Arp2/3 complex and its nucleation-promoting factors to construct branched filament networks.

Platelet plug formation needs platelets to bind

In short: Platelet plug formation needs platelets to bind to basal lamina and for platelets to pile onto each other. 2 different integrins (for collagen and fibrinogen) and 2 receptors (for ADP and thrombin) involved. Binding of platelets to Type IV collagen triggers 3 separate pathways that lead to activation of fibrinogen-binding integrin --> get platelets to interact with each other through dimeric fibrinogen.

Keratin in skin: makes the skin tough

In skin, cells express different keratin isoforms as they mature. resist pulling forces

Describe the mechanism by which the retinoblastoma protein (Rb) restricts entry into the cell cycle when appropriate nutrient levels and mitogen signals are not present

In the absence of appropriate nutrient levels and mitogen signals, Rb is bound to and inactivates the E2F transcription factor and also binds to histone deactylase which causes the chromatin to be in the closed (condensed) state Together, these activities prevent transcription of genes that encode for proteins required to enter the cell cycle

Elongation of pure microtubules

In vitro, GTP-tubulin dimers can polymerize to form microtubules Polymerization depends on the affinity of tubulin dimer to microtubule end. Tubulin dimer has higher affinity for the plus-end.

Explain why inflammatory signals cause leukocytes (aka neutrophils) that are flowing in the blood stream to start rolling on the surface of endothelial cells.

Inflammatory signals cause endothelial cells to present selectins on the cell surface— These selectins bind to mucins that are always present on the surface of leukocytes (resulting in leukocyte rolling)

DNA damage by UV irradiation frequently leads to programmed cell death (apoptosis). This effect can be mimicked by treatment of healthy cells (i.e., cells that do NOT have DNA damage) with a proteasome inhibitor called MG132. Provide an explanation for this observation

Inhibition of the proteasome prevent degradation of the p53 transcription factor—1pt Increased p53 levels lead to cell death (by inducing expression of apoptotic genes)—1pt

Binding of the hormone vasopressin to its receptor in the kidney collecting duct epithelium causes urine to become more concentrated. What is the key transporter event that happens so that water from the urine can be reabsorbed?

Insertion of aquaporins (or AQP2) into the plasma membrane

Defective integrin expression or function in neutrophils leads to a lower immune response. Provide a plausible explanation for why problems with integrin function in neutrophils can lead to decreased immune function.

Integrins are required for neutrophils to become immobilized/adhere tightly to endothelial cells near sites of injury to be able to exit blood vessels If they are unable to do this, neutrophils cannot exit blood vessels and kill bacteria, thus compromising immune function

Intermediate Filaments

Intermediate in size compared to actin and microtubules. 10 nm diameter. Flexible, strong, like ropes. Lamins oldest. Saw them in desmosomes. Important in skin, blistering diseases if mutant.

Intraflagellar transport (IFT)

Intraflagellar transport (IFT) involving additional flagellar-specific dyneins and flagellar-specific kinesins that carry packets (IFT particles) of materials out to tip (anterograde) (kinesins) where flagellum is elongating and back to base (dyneins) (retrograde) during growth and maintenance. *Existence first demonstrated with unicellular flagellate cells (like Chlamydomonas) --> tip growth of regenerating flagella.*

Gap junctions

Involved in cell-cell communication (permeable to small molecules) Permeable to molecules ≤ 800 D Sugars, amino acids, ions (electrical coupling). Also second messengers. Points that provide cytoplasmic channels from one cell to another with special membrane proteins. Also called communicating junctions.

The signal recognition particle (SRP) is a G-protein. If SRP was bound to GTPγS (a non-hydrolyzable form of GTP), how would this affect the ability of an associated ribosome to complete protein translation at the ER translocon? Explain your answer.

It would block (or slow down) protein translation because SRP would not be able to dissociate from the signal peptide.

Intercellular Junctions

Junctions are large assemblies of adhesion proteins. Epithelial cell example.

Kinesin ATPase cycle:

K alone or with ATP binds strongly to MT; K in ADP+P form binds weakly; K-ADP binds very weakly. Association with tubulin has only a modest influence on P release rate.

Kinesin ATPase and mechanical cycles

K, KTbind tightly to MT KDP binds weakly KD prone to release ADP release is rate-limiting(in absence of MTs) leading head facing the + end (2nd step in pic is the power stroke step)

Cilia and flagella beat patterns

Kartagener's syndrome Immotile cilia: - sterile : (sperm and oviduct cilia don't move) - respiratory problems: (mucus in airways can't be moved) Cilia on epithelial cells of a mussel gill

Perming

Keratin in hair is cross-linked via disulfide bonds. 1.Break existing disulfide bonds (by reducing agents) 2.Mouldhair into shape 3.Re-form disulfide bonds between keratin

. Explain why an abnormal decrease in the amount of keratin can lead to fragile skin disorders.

Keratin intermediate filaments provide mechanical strength to skin cells (because of their braided rope-like structure) and support desmosomes (or adhesion between skin cells) hence their loss of decrease tends to lead to fragile skin disorders.

In vitro motility assay for kinesin-MT

Kinesin-1 moves along a single protofilament very processively: takes multiple steps before it detaches from the microtubule. --> The movement of the two heads are coupled/ coordinated! moves cargo

Explain in general terms how ligand binding leads to activation of receptor tyrosine kinases

Ligand binding leads to receptor dimerization (homo or hetero dimerization) and transphosphorylation of the kinase domains— (transphosphorylation is the key step) Transphosphorylation gets the activation loop (or loop blocking kinase catalytic site) to move away get active receptor kinase

Five Adhesion protein classes

Ligands Cell surface proteins SelfECM proteins, ICAMs Mucins Selectins

Anchoring/linking collagens

Linking collagen (type VII): link fibrillar and sheet-forming collagens to underlying connective tissue. Defects --> blistering diseases (fragile skin) Basal lamina adheres weakly to connective tissue --> minor skin injury causes epidermis to peel from underlying dermis --> blisters (like 3rd degree burns).

Checkpoint #3: G2 DNA-repair checkpoint

Look for damage again as well as whether DNA replication and centrosome duplication are completed. When this checkpoint fails, probability is high that daughter cells will carry mutations. Checkpoint failure here is a major feature in the evolution of a tumor: the mutations can include those that create oncogenes, mutant tumor suppressors, aberrant expression of metalloproteases, etc., thereby ever increasing the capacity of the cells to lose cell-cycle control and/or to metastasize.

Centrosome maturation as cell enters mitosis

Lots more gamma-tubulin ring complexes inserted in PCM reason wanna have more motor going into mitosis: motor tips need to find kinetochore to connect to, max chance that they find target

Collagens:

Lots of different kinds of collagen. Long, stiff polymers that form a characteristic triple helix. Made and secreted primarily by fibroblasts

ECM molecules

Lots of long, fibrillar proteins with a lot of repeating motifs form an interactive network.

Adhesive glycoproteins

Lots of them. Here we will consider fibronectin and laminin. Fibronectin: Secreted by fibroblasts. Dimer. Has binding sites for cell receptors (integrins, via RGD sequence, will see later) and for collagen, proteoglycans, and other ECM molecules; *important for cell-ECM and for ECM-ECM interactions.* Basal Lamina: Continuous ECM rug under epithelia, endothelia, sleeve around muscle and peripheral nerve cells. Type IV collagen and laminin form a flat scaffold. Proteogycans present as well. NOTE: Do not confuse laminin (the cross-shaped adhesive protein in basal lamina with lamin, the intermediate filament like proteins that make up the nuclear lamina!) Matrix metalloproteases are enzymes that can breakdown the ECM (eg., the basal lamina) by cutting up ECM molecules. *Important in normal physiology (eg., shedding endometrial tissue) and for cancer metastasis.*

Microtubules with tau and MAP2

MAP2 in dendrites Tau in axons MTs bundled with tau - in axons MTs bundled with MAP2 - dendrites

Microtubule arrays

MTOC: Microtubule organizing center --> Typically centrosome Centrosomal MTs - Radial array Noncentrosomal MTs - Linear array

Alpha and beta tubulin subunits.

MTs provide support to cells and act as tracks for motor proteins (later). 25 nm diameter. Plus ends grow faster than minus, although not as big an inherent differential as barbed/pointed in actin.

Many cellular processes are targeted by pathogens for their benefit. Discuss one specific example of a pathogen and how it exploits a cellular process to its advantage

Many possible examples; only some listed below—need to name pathogen (1pt) and state the cellular process that the pathogen takes advantage of (2pts) • Listeria—lyse phagosome membrane or recruits actin assembly to move inside cell • TB bacteria—inhibits lysosome fusion with phagosome • Diphtheria bacteria—get toxin (DTA) out of endosomes to kill cells • Cholera bacteria—keeps Gα active longer, leading to overproduction of cAMP (and Cl- efflux) • Influenza virus—fuse with endosome membrane to escape • HIV—kill helper T-cells or suppress helper T-cell activation

The ECM is the part of our body that is not cells. Important for reasons such as:

Mechanical support of cells •Signaling: control of cell division •Scaffolding for tissue organization •Substrate for cell adhesion and motility

non polar and hydrophobic and uncharged amino acids MAIL F V W

Methionine (Met) Alanine (Ala) Isoleucine (Ile) Leucine (Leu) Phenylalanine (Phe); Valine (Val); Tryptophan (Trp);

Comparing cytoskeleton filaments (enlarged by 10^6):

Microtubules: Like pipes. Stiff, resist compression, best choice for asymmetrical structures (because they don't bend easily). Actin: Like rods. Bend much more easily than microtubules. Need to bundled/crosslinked to stabilize structures (eg., microvilli) in cells. Intermediate filaments: Like braided ropes. Flexible but very strong. Provide strength to cells. Subunits : Different from actin and tubulin in that subunits vary a lot in size and sequence, but all have central coiled-coil domain. Globular N- and C- termini give special features.

Telophase

Mitotic cyclins destroyed by now, cell reverses to interphase state. Lamins lose phosphates and reassemble, Nuclear envelopes form around separated daughters, microtubules return to interphase behavior, position of cleavage furrow specified (not responsible for signals regulating position of cleave furrow) and necessary proteins move into the midline.

You discover a new protein that transports sucrose against its concentration gradient into cells. You later find out that this protein does NOT hydrolyze ATP. Provide a possible explanation for how this transporter is able to import sucrose against its concentration gradient without itself using ATP as the energy source.

Most likely this new transporter is a carrier protein that uses the energy from an ion gradient generated by a pump to transport sucrose into the cell. (Picture showing this as part of a chemiosmotic cycle is also fine).

Motors have directionality: (polarity of track becomes important) - myosin -kinesins -dyneins

Most myosins walk towards *barbed* end of actin - except Myosin VI--> pointed end Most kinesins walk towards *plus* end of MT (anterograde) - except Kinesin 14 -->minus end Dyneins walk primarily towards *minus* end of MT (retrograde)

If kinesin has: Motor domain at N-terminus --> kinesin 5 Motor domain at C-terminus --> kinesin 14 Motor domain internal (middle of the protein) --> kinesin 13

Motor domain at N-terminus --> kinesin moves towards microtubule plus end ---- kinesin 5 Motor domain at C-terminus --> kinesin moves towards microtubule minus end-- Motor domain internal (middle of the protein) --> kinesin depolymerizes microtubules

Locomotion by cilia and flagella: Cilia/Flagella:

Move organism or, if cell tethered, move fluids (e.g. trachea and bronchi).

Briefly describe the classical experiment that led to the "pointed" end and "barbed" end terminology for actin filament polarity.

Myosin --> digest with protease --> get myosin head and incubate with actin filaments --> image using EM (1pt upto now)--> see "arrow head" type decoration--> pointed end and barbed end (1pt for proper positioning of the ends).

In vitro motility assay ("gliding assay") for actin/myosin

Myosin attached to glass Actin filaments fluorescent •Actin filaments move unidirectionally •For MyosinII, actin moves with pointed end leading

Refer to the images below for the following questions. Actin decoration by myosin II Microtubule decoration by kinesin-1 a) For these decoration experiments to work, both myosin-II and kinesin-1 have to be in particular nucleotide states. State the nucleotide state and explain its importance for successful decoration. b) If these motors were used for gliding assays, which end of the actin or microtubule filament would be leading and why?

Myosin-II decoration - Nucleotide state : No nucleotide - Reason : The empty head state is the rigor state for myosin-II which allows it to bind stably to actin Kinesin-1 decoration - Nucleotide state : ATP (or ATPγS or AMP-PNP) - Reason : ATP bound to the head is the rigor state causing kinesin to bind stably to microtubules Myosin-II gliding assay: Pointed-end leading Kinesin-1 gliding assay: Minus-end leading

Motility by pseudopod extension

Myosin-based retraction of tail Pseudopod leading edge: actin assembly and disassembly --> extension Contribution of actin crosslinking to pseudopod extension Dendritic actin model at pseudopod leading edge F- actin generates force to cause membrane extension

Cytoskeleton: structural role and as tracks(actin and MTs)

Myosin: Actin-based motors Kinesin and Dynein: MT-based motors --- Myosins and Kinesins derive from primordial NTPase fold resembling GTPases. dynein- entirely different class of proteins --- Dynein a different evolutionary idea (AAA ATPase)

Intracellular movement

Myosins drive cytoplasmic streaming (faster known motors- speed of 20 μm/s!) (kinesin-1 speed is about 0.4 μm/s) Most cargos have all types of motors on them: something must coordinate motor activity...

The cytoskeleton: A very dynamic structure!! What is it?

Network of filamentous protein polymers (actin filaments, microtubules and intermediate filaments)

State a specific molecular event during neutrophil rolling & migration and platelet activation & aggregation that illustrates the following principles of cell adhesion Principle of cell adhesion: Binding preferences for cell adhesion proteins Regulation of the activation state of cell adhesion proteins

Neutrophil rolling/migration Selectins on endothelial cells bind to neutrophil mucins (or vice versa) Integrins on endothelial cells activated by chemoattractants (or N-formyl peptides) Platelet activation/aggregation Collagen integrins on platelets bind to collagen in basal lamina OR fibrinogen integrins bind to fibrinogen [after activation] Fibrinogen integrins activated by binding of platelet to collagen OR Fibrinogen integrins activated by ADP signaling pathway OR Fibrinogen integrins activated by thrombin signaling pathway

Neutrophil capture and migration

Neutrophils are flowing rapidly in the blood stream. Need a way to slow them down, then immobilize them in face of blood flow at sites of infection (to allow their emigration).

White Blood Cells: Granulocytes: Neutrophils

Neutrophils or Polymorphonucleoleukocytes = "Polys" •80g/day in bone marrow •Find and destroy bacteria (first line of defense) •Attracted by N-formylated peptides --> chemotaxis •Kill bacteria by phagocytosis and superoxides

Basal bodies and cilia.

Nine doublet microtubules at periphery with a central MT pair. The doublet MTs are way MORE stable than cytoplasmic microtubules.

Prometaphase - steps

Nuclear envelope breakdown via Cdk1/cyclin B phosphorylation of lamins (don't worry about all the ambiguities voiced in text). Steps, Lamin B phosphorylation--> monomers--> remain associated with nuclear envelope vesicles because of farnesyl group. Lamins A and C disassemble into monomers. Nuclear envelope breakdown enables the chromosomes to interact with microtubules.

What allows actin severing proteins (eg., cofilin) to preferentially target older actin filaments?

Older actin filaments are distinguished by ADP-actin

Give one example each of an oncogene and a tumor suppressor gene. Which one is dominant and which one is recessive? Please explain your answer

Oncogene: Ras, EGF receptor, many others Tumor Suppressor: Rb, p53 Oncogene is dominant because only one bad copy is sufficient to inappropriately stimulate cell division Tumor suppressor is recessive because only one bad copy cannot prevent inhibition of entry into cell cycle (i.e., one good copy will still do the job), both copies need to be mutated to progress into the cell cycle

Restriction point and cancer

Oncogene: genes whose inappropriate *activation* leads to cancer (Ras, RTK receptors). Stuck doorbell --> *dominant*. Rb is called a "*tumor suppressor*": keep cell arrested in G1 - gene whose *inactivation* leads to cancer. If mutant, appropriate keep E2F in check. (Rb blocks expression of DNA replication genes (and others) by inhibiting transcription factor E2F (job to express genes important for S phase- DNA replication).) *Recessive* because one normal copy can do the job. Cells lacking Rb function or with active oncogenes can pass the restriction point inappropriately

Oncogenes vs. tumor suppressors

Oncogenes: inappropriate activation is the problem. Dominant (genetically); mutant Ras, tyrosine kinase receptors, etc... Tumor suppressors: inappropriate inactivation is the problem. Recessive, because one good copy is enough. Rb and p53 as examples.

Integrin conformations influence affinity

Open state can be generated by the binding of a ligand to the head or by a signaling protein binding to the cytoplasmic tail

if you performed affinity chromatography using beads coupled to an SH2 domain, what kinds of ligands would you expect to bind to these beads?

Phospho-tyrosines

Platelet activation, aggregation to repair damaged endothelium For platelet plug formation, need platelets t

Platelet activation and aggregation --> prevent bleeding. But inappropriate activation --> blood clots leading to heart attacks and strokes For platelet plug formation, need platelets to: 1) bind to basal lamina 2) bind to one another

Platelets carry 7-helix receptors for

Platelets carry 7-helix receptors for *thrombin and ADP*

+TIP proteins (eg. EB1) associate with the plus end of growing microtubules

Plus-end tracking proteins (+TIPs) •Regulate MT plus-end dynamics •Link MT plus ends to other structures (eg., plasma membrane, actin, vesicles, chromosomes) •EB1 "finds" the growing MT plus-ends via the GTP cap structure

Generic collagen triple helix

Predominant connective tissue protein; secreted by fibroblasts Gly-X-Y sequence; usually X=proline, Y= hydroxyproline Polyprolinehelix (also seen in ligands of SH3 domains) 3 chains form triple helix

What is the sequence of steps that Terry Deacon proposes to get from primal soup to an autocell. Use the following terms in your answer: Container formation, primal soup, autocatalytic cycle and transient container disruption.

Primal soup --> Autocatalytic cycle --> Container formation --> Transient container disruption

Primary cilium (9 + 0 axoneme) on mesenchyme cell

Primary cilia are non-motile! (9 + 0 axoneme) They typically serve a sensory function (eg., olfactory and vision neuron) Mutations affecting IFT --> defective primary cilia --> blindness, polycystic kidney diseases (affect 600,000 Americans: cysts --> destruction of nephrons)

Lipids and H+ gradients play a crucial role in cells. Describe their involvement in any one of the two processes listed below. Function of lipids: (a) Protein sorting in secretory pathway OR (b) Signaling Function of protons (H+): (a) Protein degradation OR (b) Chemiosmotic cycle

Protein sorting: Phospholipids and sphingolipids+cholesterol membranes have different thicknesses. This helps to sort transmembrane proteins according to the length of their transmembrane alpha helix. Signaling: Lipids serve as substrates for kinases or phosphatases or lipases to generate second messengers that activate downstream targets. Protein degradation: Low pH of the lysosome is important to activate the lytic enzymes that degrade proteins Chemiosmotic cycle: pH gradient (or proton gradient) can drive uptake of solutes (e.g., neurotransmitters) into organelles (or vesicles) through antiporters or symporters OR proton gradient across inner mitochondrial membrane drives ATP synthesis by moving through the F-type ATP synthase.

Why is the rate of transport through channels significantly faster than through pumps?

Pumps have to hydrolyze ATP and bind and unbind their ion/solute In contrast, channels simply passively conduct ions through their pore , hence they are faster.

Integrins: main cellular receptors for ECM and other proteins on cells

RGD= integrin binding site on 1/3 of ECM ligands "Integrin activation": ligand binding activates integrins --> signals Promiscuous. Integrins can bind to multiple ligands and multiple ligands can bind to a single integrin. e.g. fibronectin'sRGDbinds to 9 different integrins 18 alphas, 8 betas, alternative splicing --> 128 possible dimers Ligands bind to I-domains and beta propeller

Morphology and distribution of microtubules:

Radial organization around centrosome in interphase; Bipolar (astral) spindle in mitosis --> centrosomal microtubules •Linear bundles along sides of columnar epithelial cells. Not attached to centrosome --> noncentrosomal microtubules

Regulation of restriction-point passage by retinoblastoma protein (pRb, aka Rb)

Rb blocks expression of DNA replication genes (and others) by inhibiting transcription factor E2F (job to express genes important for S phase- DNA replication). also deactylates histone: make histones more tightly compacted; cannot be replicated Growth factor (or related forms of signaling) --> Cdk 4/6-D activated --> pRb phosphorylated and dissociates --> genes expressed --> cross restriction point --> S-phase Acetylation of histones favors an "open" chromatin --> better for gene transcription

Matrix metalloproteinases (MMP)

Remodel the ECM (many have lysin name because they "lyse" or fragment tissue) Propeptide cleaved to activate; Need metal ions like Zn2+ Involved in: Menstruation (shedding of endometrium) Metastases

Cell cycle phases: G1, S, G2, M. S phase identified as period that cells are labeled with

S phase identified as period that cells are labeled with radioactive thymidine=DNA replication; two gaps (G1 and G2) between S phase and mitosis.

last checkpoint: Spindle assembly checkpoint (SAC) checkpoint Failure --> aneuploidy (abnormal chromosome number)

SAC is "on" by default --> turned off when chromosomes are properly attached to the spindle Target of SAC is APC/C (Cdc20) - key activator for APC, w/o it APC is inactive -- NEED at activate APC to enter anaphase Free kinetochores enable Mad2 protein to bind to Cdc20 (which is needed to activate APC/C)

Give an example of secondary and quaternary protein structure from this course. (1pt each = 2pts total) Secondary structure: Quaternary structure

Secondary structure: alpha-helix OR beta-sheet OR coiled-coil Quaternary structure: Lots of possible examples! S5-P-S6 channels, many pumps, many carriers, heterotrimeric G-proteins, etc. etc. Any one example is fine. [NOTE: macromolecular assembly like clathrin coat, F-actin and MTs are NOT examples of quaternary structure]

Fibronectin (adhesive ECM protein)

Secreted by fibroblasts. Help connect cells to the ECM RGD sequence binds to integrins on cell surface Other adhesive proteins --laminin, tenascin, von Wille brand's factor, many more --link cells to matrix and matrix to matrix

Selectins(lectins) and mucins

Selectins have a lectin domain that binds to the sugars on mucins. Expressed on white blood cells and endothelial cells

polar uncharged amino acids STQNY*

Serine (Ser) Threonine (Thr) Glutamine (Gln) Asparagine (Asn) Tyrosine (Tyr); polar and uncharged and *hydrophobic*

Katanin:

Severing protein. AAA ATPase protein. Assembles on microtubules sides as a hexamer --> uses ATP hydrolysis to cut microtubules

metaphase; Spindle checkpoint

Signaling by unattached kinetochores keeps cells from entering anaphase. Enter anaphase due to *APC/C (E3) destroying securin*; therefore, prevent going into anaphase by inhibiting APC/C. This is done by the protein Mad1, which binds to kinetochore coronas that don't have attached microtubules. The Mad1 activates Mad2, which in turn binds to Cdc20, the protein needed to activate to APC/C --> APC/C inactive. When MTs bind to kinetochores, Mad1 is kicked off and can't activate Mad2--> Cdc20 available to activate APC/C--> enter anaphase. This checkpoint crucial since if cells override it and divide anyway, you get aneuploid daughter cells with too many or too few chromosomes. A 2n-1 cell can express recessive genes on the single homologue, including, e.g. mutant tumor suppressors, thereby abetting the march to malignancy; many tumors are aneuploid

Cell cycles in skin

Skin: cell division --> differentiation -->terminal differentiation --> cell death

Keratin mutations and blistering diseases

Spinous layer cells die too early Hyperkeratosis - Excess scaling of skin

(MAPs) *Microtubule associated proteins* = those to remember

Stabilizing MAPs *Tau --> Binds side and stabilizes microtubules....... MAP2 helps maintain spacing of bundle MT Destabilizing MAPs *Katanin --> AAA ATPase that severs microtubules - kinesin 13* End-binding proteins (+TIPs) *EB1 --> Bind to (+) ends, reduce catastrophies, link MTs to other cellular structures

Microtubules structure

Stiff, noncompressible cylindrical polymers - pipe like, hollow

Major molecules that are part of the ECM

Structural proteins: collagen and elastin Proteoglycans: gel-like polymers, hydrate the ECM Adhesive glycoproteins: fibronectin and laminin that enable cell-ECM and ECM-ECM interactions

Actin monomer: G-(globular) actin

Structure very highly conserved; has polarity Pointed end - cytoplasm facing Barbed end - membrane facing

Tubulin and microtubule structure

Subunits assemble in a head-to-tail manner. β-tubulin faces plus end, α- tubulin faces minus end In MT polymer, longitudinal and lateral contacts between tubulin dimers 13 protofilaments

Glycosaminoglycans(GAGs) and proteoglycans

Sugars in GAGs usually uronic acid and hexosamine, sulfated •Usually attached to proteins --> proteoglycans (glycan part dominates) •Exception: hyaluronan/hyaluronic acid (not attached to a protein) •Proteoglycans made by all cell types; major constituents of cartilage, loose connective tissue, and basal lamina •Polyanions --> extended by repulsion, attract water, large volume filled •Serve as lubricants in joints

Why does the barbed end of an actin filament polymerize faster than the pointed end?

The barbed end has a much lower critical concentration for G-actin than the pointed end.

Tubulin has a "curved" conformation upon microtubule formation

The curved conformation adds strain on the microtubule lattice --> destabilizing

Mast cells are key players in initiating allergic responses. a) What is the mechanism by which mast cells specifically recognize particular antigens? b) How does recognition of antigens by mast cells lead to allergy symptoms?

The mast cells have receptors for the Fc part of IgE molecules that recognize specific antigens Antigen recognition leads to secretion of histamine from mast cells, which binds to receptors on target cells resulting in allergy symptoms.

Intercellular junctions

The mechanical integrity of tissues depends on cell-cell adhesion. Intercellular junctions are strucutres between cells that allow neighboring cells to form strong connections with each other, prevent passage of materials, or establish rapid communication between adjacent cells. The three types of intercellular contact in animal cells are: desmosomes, gap junctions, and tight junctions.

The extracellular matrix

The noncellular material of tissues in which cells live and to which cells bind Part of the tissue that is not a cell

Briefly describe how a pseudo-substrate motif regulates protein kinase C (PKC) activity.

The pseudo-substrate motif resembles a substrate site and binds to PKC. However, it cannot be phosphorylated (because it lacks a S/T residue) and hence inhibits the catalytic (kinase) activity of PKC.

. Refer to the cell sorting experiment below for the following questions. a) At the beginning of this experiment, a chelator was used to take away Ca2+ and induce cell dissociation. Explain why removal of Ca2+ caused the cells to dissociate b) What general principle of cell adhesion is illustrated by this experiment? Explain your answer

These cells interact through Cadherins which require Ca to be able to bind to each other b) What general principle of cell adhesion is illustrated by this experiment? Explain your answer. (4pts) Principle: Expression level or surface density of adhesion proteins regulates cell adhesion— In this experiment, cells with high cadherin levels tend to stick first, then the cells with low levels of cadherin

Families of actin-binding proteins: Crosslinking

These proteins bundle and/or crosslink actin filaments. Regulate spacing within actin bundles Actin bundle formed by fimbrin (denser) Actin bundle formed by α-actinin

Why do certain snake venoms inhibit blood clotting?

These snake venoms have disintegrins/proteins that have RGD motifs which bind to integrins on platelets and inhibit clot formation. Some snake venoms contain proteins that have RGD motifs (called disintegrins) These compete with fibrinogen for the integrin receptor and thus platelets cannot form a clot

F-actin cross-linking proteins

They have two actin binding sites. Random networks or more regular bundles created. Of those named,remember: α-actinin: generally in random, open-type actin networks Fimbrin: in microvilli, close regular actin bundles Spectrin: already seen in RBC, also binds, via linkers, to membrane proteins like Band3

Think about the various mechanisms that regulate the activity of cyclin-dependent kinases (Cdks). a) Why would a mutation that changed the threonine residue that is phosphorylated by CAK to alanine cause cells to be bigger than wild type cells? b) Why would overexpression of Cdc25 cause cells to be smaller than wild type cells?

This mutation would prevent maximal activation of Cdks thus inhibiting mitosis and causing the cells to keep growing (making them bigger than wild-type cells) b) Why would overexpression of Cdc25 cause cells to be smaller than wild type cells? (2pts) Overexpression of Cdc25 would cause Cdks to become maximally active all the time (or at least more of the time) pushing cells into mitosis and making them smaller.

What is the function of check point systems in the cell cycle?

To detect problems (eg., DNA damage) and arrest the cell cycle to allow for repair OR detect problems and induce cell death if the problem is not fixed in time (either answer is fine)

F-actin nucleation, growth, and nucleotide exchange in vitro

Trimer= nucleus for elongation Old actin filament distinguished by ADP-actin

The thin filament in sarcomeres consists of actin plus actin-binding proteins. The protein that caps the pointed-end of actin in thin filaments is called: The thin filament protein that responds to Ca2+ release in the cytoplasm is called:

Tropomodulin Troponin-C

Hooking cycle up with motion:

Two heads influence each other: one binds tightly, the other binds loosely. ATP binding to the leading head causes the neck-linker to zipper along the head causing the lagging head (that's loosely bound or unbound to microtubules) to swing forward.

myosin V transporting vesicles along actin filaments from mother cell to daughter bud in budding yeast (S. cerevisiae)

Unlike Myosin II, Myosin V moves processively •Myosin V is a processive motor (unlike myosin II) •Myosin V takes 36 nm steps, which matches the helical pitch of actin filaments (so, the motor stays on the same side of the filament) Vesicles etc. can also be transported along actin tracks via myosin V (a processive myosin unlike Myosin II) to achieve intracellular transport.

Intermediate filament proteins - subunits

Unlike actin and tubulin, subunits vary in size Typically particular intermediate filaments are expressed in different cell types (can be important in tumor diagnosis)

How motors work Basic concept of motor protein

Use ATP hydrolysis to produce force/motion (conformational change) Basic steps 1.Grab 2.Pull 3.Release and repeat motor/head domain: ATP hydrolysis and bind track tail domain: bind cargo •Mechanochemical enzyme: energy from ATP hydrolysis drives conformational change --> movement. •Head/Motor domain: binds track and hydrolyzes ATP •Tail domain: binds cargo (or itself as in myosin II) •If motor anchored, polymer moves; if polymer anchored, motor + cargo moves.

Dynamic instability in vitro:

Use axoneme to "seed" microtubule polymerization. See phases of growth and shortening that randomly switch from one to the other This behavior is driven by GTP hydrolysis at exchangeable site on beta-tubulin (GTP in alpha stays GTP). GTP-tubulin --> straight conformation --> stable MT filament GDP-tubulin --> curved conformation --> stress of filament, liable to peel off"GTP caps" at ends protect the microtubule from depolymerizing. If cap lost --> have "catastrophe" i.e., shortening because GDP polymer is so unstable.

Actin-like genes arose in the common ancestor & are found in all three domains of life

Used to think that cytoskeleton was unique to eukaryotes Actin in bacteria •cell shape (rods) •Segregate plasmids

Movement of Cells

Velocities of moving cells vary by > 4 orders of magnitude

Tubulin heterodimer

Very stable association of alpha-beta tubulin (like beta-gamma subunit of G protein). Alpha-tubulin GTP non-exchangeable, beta-tubulin GTP exchangeable, important for dynamic instability of polymer Plus and minus end, analogous to actin barbed and pointed. *Protofilaments*, usually 13 protofilaments per microtubule. Protofilaments have polarity due to orientation of dimers: beta-tubulin oriented towards plus end, alpha-tubulin towards minus. Tubulin gene family. Alpha-beta polymerize, gamma-tubulin in nucleation complex. Bacterial ancestor = FtsZ

Dynamic instability of microtubules in vitro

Whereas F-actin *will grow continuously* as long there is sufficient G-actin around, a steadily growing MT can suddenly shrink *even when* there is ample tubulin around! •Tubulin polymerization: β-tubulin GTP -->GDP •GDP polymer under stress (due to "curved" tubulin) •GTP cap at plus-end stabilizes it. •GTP-cap loss --> catastrophe

Mutant CFTR (Cystic Fibrosis Transmembrane Regulator) proteins frequently have trouble folding correctly and get degraded by the ERAD pathway, thus leading to low levels of CFTR protein in the plasma membrane of lung epithelia. Imagine that you did an experiment in which you prevented N-linked glycosylation of one such mutant CFTR protein and found that this modified version of CFTR is now present in the plasma membrane at high levels. Provide a plausible explanation for this observation.

Without N-linked glycosylation, the mutant CFTR would not cut stuck in the Calnexin cycle and hence would not be prone to ERAD.

FtsZ rings

a bacterial microtubulin version Forms ring structures that drive cell division

Dynactin

a dynein complex that allows vesicles to move on MTs. "Actin" part of dynactin is actually actin-related protein Arp1 (the other Arp besides Arp2/3 associated with actin polymerization) and there's a protein called p150 glued that links dynactin to dynein. Dynactin binds to spectrin around vesicle and recruits dynein to vesicle for retrograde transport.

Regulation is a central theme in cell biology. a) Describe one way by which entry of a transcription factor into the nucleus is regulated. The specific names in [brackets] below are not needed. b) Describe one way by which opening of an ion channel is regulated

a) One of the following examples: • Transcription factors in the cytoplasm may be phosphorylated [e.g., STATs, Smads] OR dephosphorylated [e.g., NF-AT] • Blocked/masked NLS can be exposed [e.g., NF-kB] • Transcription factor may be untethered [e.g., dorsal] b) One of the following ways: • Change in membrane potential (or voltage) • Intracellular ligands (e.g., cyclic nucleotides) • Extracellular ligands (e.g., neurotransmitters, IP3, etc) • Conformational change in interacting protein (e.g., ryanodine receptor-DHP) • Phosphorylation • Change in temperature (or binding of menthol or capsaisin)

State the directionality of the motor protein based on the gliding assay results listed below. a) Actin filaments gliding with their barbed ends leading b) Microtubules gliding with their plus-ends leading

a) Actin filaments gliding with their barbed ends leading : Pointed-end directed motor b) Microtubules gliding with their plus-ends leading : Minus-end directed motor

Floppy or uncoordinated muscle contraction typically leads to muscle loss over time. Briefly state the activity of the following proteins that contributes to proper muscle contraction. (2pts each = 6pts total) a) CapZ capping protein: b) Desmin: c) Dystrophin:

a) CapZ capping protein: Cap barbed-end of actin filaments to help keep them at a fixed length b) Desmin: Link Z-disks between neighboring myofibrils to laterally align sarcomeres c) Dystrophin: Link internal cytoskeleton (or actin cytoskeleton) to basal lamina to withstand forceful contractions

MG-132 is a drug that specifically inhibits the 26S proteasome. How would this drug affect the situations listed below? a) Cyclin B protein level at anaphase b) Misfolded proteins in the ER: c) MHC type II receptor-peptide complex formation in macrophage:

a) Cyclin B protein level at anaphase: CyclinB would not be degraded OR cyclin B levels would be steady and not drop. b) Misfolded proteins in the ER: Misfolded proteins would accumulate in the cell because their degradation requires the proteasome [as part of ERAD pathway] c) MHC type II receptor-peptide complex formation in macrophage: This process would be unaffected because it does not rely on the proteasome. [happens in the lysosome]

State whether the structures or processes below represent thermodynamics, morphodynamics or teleodynamics. a) DNA serves as a system of instructions: b) Binding of enzyme and substrate: c) Negative charges repel each other: d) The arrangement of gold atoms in a piece of jewelry:

a) DNA serves as a system of instructions: teleodynamics b) Binding of enzyme and substrate: morphodynamics c) Negative charges repel each other: thermodynamics d) The arrangement of gold atoms in a piece of jewelry: morphodynamics

What activity/ability would the below proteins lose after the stated deletions? a) Delete RGD motifs from Fibronectin: b) Delete the actin-binding domain on Dystrophin: c) Delete profilin binding sites on Formin: d) Delete γ-tubulin from γ-tubulin ring complexes: e) Delete the oligosaccharides on mucins: f) Delete the Ca2+-binding sites on cadherin:

a) Delete RGD motifs from Fibronectin: Prevent fibronectin from binding to (specific) integrins b) Delete the actin-binding domain on Dystrophin: Dystrophin cannot link actin cytoskeleton to basal lamina c) Delete profilin binding sites on Formin: Inhibit actin nucleation/polymerization by formin d) Delete γ-tubulin from γ-tubulin ring complexes: Prevent nucleation of microtubules e) Delete the oligosaccharides on mucins: Prevent selectins from binding f) Delete the Ca2+-binding sites on cadherin: Make cadherins floppy OR prevent cadherins from interacting with each other

What activity/ability would the proteins below lose following the stated deletions? (1pt each = 6pts total) a) Delete stalk domain from dynein: b) Delete the transmembrane α-helix from an Ig-CAM: c) Delete the N-terminal projection domain from MAP2 d) Delete the actin binding domain on WASP: e) Delete the ATP binding site on katanin: f) Delete the tail domain of kinesin-1:

a) Delete stalk domain from dynein: binding to microtubules b) Delete the transmembrane α-helix from an Ig-CAM: mediate cell adhesion or be present in the PM c) Delete the N-terminal projection domain from MAP2: spacing between microtubules (would be lot less) d) Delete the actin binding domain on WASP: activate Arp2/3 complex or produce branched actin filaments e) Delete the ATP binding site on katanin: hydrolyze ATP or sever microtubules f) Delete the tail domain of kinesin-1: bind to cargo or transport cargo

State one function of the following microtubule-associated proteins? a) End-Binding 1 (EB1): b) Katanin:

a) End-Binding 1 (EB1): Stabilize microtubules plus-ends (promote plus-end polymerization) OR mediate binding of microtubules plus-ends to other cellular structures (eg., kinetochores, PM) b) Katanin: Sever or cut microtubules (cut microtubules may depolymerize to decrease MT mass or repolymerize/rescue to increase microtubule mass)

. Decades before the protein constituents of muscle were known, biophysicists studied the mechanical properties of isolated whole muscle. The graph below plots the force and power of muscle as a function of muscle shortening velocity (i.e., rate of contraction) as described by the Hill equation. Use this information to answer the following questions. a) Why can't you lift a piano very quickly but can tap on its keys at a much faster rate? b) Notice that muscle does not generate power under two conditions (circles A and B in the graph). What is the sarcomere contraction state under these two conditions? Condition A Condition B:

a) High force production is only possible when muscle contraction is slow Conversely, muscle produces greatest speed under no load (or under light load) b) Notice that muscle does not generate power under two conditions (circles A and B in the graph). What is the sarcomere contraction state under these two conditions? Condition A: Relaxed state Condition B: Contracted state [but under no load meaning muscles that are contracting as fast as they can produce no power when under zero load]

The sarcomere is the most basic contractile unit of muscle. a) If the polarity of actin filaments was reversed in a sarcomere, explain how the movement of myosin-II on these actin filaments would affect sarcomere length b) Why are actin filaments stable (i.e., of defined length) in muscle cells?

a) If the polarity of actin filaments was reversed in a sarcomere, explain how the movement of myosin-II on these actin filaments would affect sarcomere length. Sarcomere length would increase because myosin-II moves towards the barbed end, which would work to move the actin filaments apart in this case. b) Why are actin filaments stable (i.e., of defined length) in muscle cells? Because of capping proteins at the barbed and pointed ends. (not necessary to name the capping proteins—which are CapZ and tropomodulin, but fine if done)

A balance of forces between opposing motor proteins is essential for the proper size and function of the spindle apparatus. a) If you decreased kinesin-13 activity, how would this affect spindle length? Explain your answer b) What other motor activity could you change to restore a balance of forces following the change listed above? Name the type of motor and state how its altered activity would work to restore a balance of forces

a) If you decreased kinesin-13 activity, how would this affect spindle length? Explain your answer. (2pts) Spindle would lengthen because microtubule depolymerization from the poles is reduced b) What other motor activity could you change to restore a balance of forces following the change listed above? Name the type of motor and state how its altered activity would work to restore a balance of forces. (2pts) Any one of the following answers: • Either increase kinesin-14 activity to enhance inward sliding of interpolar MTs • Decrease kinesin-5 activity to reduce outward sliding of interpolar MTs • Decrease cytoplasmic dynein activity to reduce pulling (or outward) forces on radial MTs

State whether the treatments below would increase or decrease muscle contraction? Explain why. a) Inhibition of the Ca2+ pump in the sarcoplasmic reticulum b) Inhibition of the Ca2+ channel in the sarcoplasmic reticulum

a) Inhibition of the Ca2+ pump in the sarcoplasmic reticulum (1pt) Would increase muscle contraction because Ca would persist in the cytoplasm thus keeping myosin binding sites exposed for longer times. b) Inhibition of the Ca2+ channel in the sarcoplasmic reticulum (1pt) Would decrease muscle contraction because myosin binding sites would not be exposed (since Troponin-C would not be active)

G-proteins are master regulators of numerous cellular processes. For each of the processes listed below, name one applicable G-protein and briefly describe its function in the stated process. G-protein: Function: for a) Membrane trafficking b) Nuclear transport c) Signal transduction

a) Membrane trafficking (lots of possible answers—any one G-protein and its correct function is fine) G-protein: Possible answers are: Sar1, Arf, Rab, dynamin Function: Sar1 and Arf function—bind to coat proteins (or effectors) OR sort appropriate vescicular cargo; Rab function—bind to SNAREs and tethers OR confer membrane identity; dynamin function—membrane scission. b) Nuclear transport G-protein: Ran Function: Directionality of nucleocytoplasmic transport OR confer nucleus versus cytoplasm identity OR facilitate nuclear import or export c) Signal transduction (lots of possible answers—any one G-protein and its correct function is fine) G-protein: Possible answers are: Gα, Gsα, Giα, Gt (transducin), Gαβγ (or heterotrimeric G-protein), Ras Function: For any of the heterotrimeric G-protein components, the function is either activate adenylyl cyclase, inhibit adenylyl cyclase, activate cGMP phosphodiesterase as appropriate. For Ras, function is activate MAPKKK (or Raf)

The cytoskeleton has multiple functions in a cell. State in one sentence how the cytoskeleton contributes to the cellular activities listed below. a) Movement of integral membrane proteins within the plasma membrane: b) Trafficking of secretory vesicles: c) Cytokinesis: d) Cell adhesion:

a) Movement of integral membrane proteins within the plasma membrane: Cytoskeleton may restrict or corral membrane proteins OR membrane proteins may be transported along cytoskeletal filaments b) Trafficking of secretory vesicles: Transport of vesicles along cytoskeleton c) Cytokinesis: Actin-myosin constricts the plasma membrane (at the cell division site) d) Cell adhesion: Cytoskeleton supports cell adhesion proteins/sites OR cytoskeleton can mediate signaling at cell adhesion sites

Like many kinases, Cdk1 activity is tightly regulated by cells. For each of the regulatory steps listed below, name the protein involved and state its effect on Cdk1 kinase activity a) Non kinase partner b) Two kinases c) Phosphatase ^^ all of their effects in cdk1 kinase activity

a) Non kinase partner: CyclinB --> Binds to Cdk1; get basal activity b) Two kinases: A) CAK --> Phosphorylates Cdk1 (at T160); increases activity (to max level) B) Wee1 --> Further phosphorylates Cdk1; inactivates Cdk1 (or greatly reduces activity) c) Phosphatase: Cdc25 --> Removes the phosphates added by Wee1; get back to max activity

The fibrinogen-binding integrin plays a key role in platelet aggregation. Explain one way by which this integrin is activated in platelets stuck to the site of injury and in nearby platelets in the blood stream. a) One mechanism of integrin activation in an adhered platelet b) One mechanism of integrin activation in platelets that are in the blood stream.

a) One mechanism of integrin activation in an adhered platelet. One of the following answers: i) Interaction of collagen integrin with collagen (in basal lamina) leads to a signaling pathway that activates the fibrinogen integrin in these cells. ii) Thrombin activates its receptor which leads to activation of the fibrinogen integrin. b) One mechanism of integrin activation in platelets that are in the blood stream. One of the following answers: i) ADP released by platelets adhered to the basal lamina binds to receptors on platelets and leads to activation of their fibrinogen integrins. ii) Thrombin activates its receptor which leads to activation of the fibrinogen integrin.

The restriction point in the G1 phase of the cell cycle monitors if conditions are suitable for cell division and prevents cells from dividing at inappropriate times. a) Name the key protein that enforces the restriction point: b) State the two mechanisms by which this protein prevents expression of genes required for S-phase c) How do appropriate growth factors relieve this inhibition and allow progression into S-phase?

a) Retinoblastoma or pRb or Rb__ b) Rb binds to the transcription factor E2F, which prevents E2F from recruiting gene transcription apparatus (RNA polymerase etc.)—1pt Rb gets histone deacetylase to the chromatin, which results in compact chromatin structure which prevents gene expression—1pt c) Growth factors result in synthesis of Cyclin D— This leads to Cdk4/6 activation—1pt Which phosphorylates Rb, thus relieving inhibition of gene expression

For each of the aims stated below, name a technique that will give you the desired information. a) Determine the molecular weight of a protein: Any one of the following: b) Determine if gene A is expressed in muscle cells: Any one of the following: c) Determine if protein B is found in the nucleus

a) SDS-PAGE, western blot, mass-spectrometry, gel filtration b) Any one of the following: Northern blot, RT-PCR, microarray, RNA-Seq__ c) Immunostain or immunofluorescence, immunogold, GFP technology, cell fractionation

Thinking about transport across membranes. a) Steroid sex hormones such as progesterone and testosterone are synthesized from cholesterol. However, while steroid hormones enter target cells by passive diffusion, LDL particles that contain cholesterol require the LDL receptor to get into cells. What is the reason for this difference? b) Why is it that pumps can generate ion gradients on their own whereas carriers and channels cannot produce ion gradients on their own?

a) Steroid hormones are hydrophobic and can freely diffuse through membranes. However, the LDL particle consists of (esterified) cholesterol and a large protein (ApoB), which requires endocytosis to get into the cell. b) Pumps use energy from ATP hydrolysis (or light) to move ions against a gradient. In contrast, carriers and channels by themselves do not consume energy and hence cannot create ion gradients.

State how autocells and real cells accomplish the following tasks: a) Uptake of nutrients/substrates b) Generation of a new metabolic product

a) Uptake of nutrients/substrates Autocell: (transient) disruption of container Real cell: uptake through transport proteins (such as channels, carriers and pumps) or endocytosis (1pt) b) Generation of a new metabolic product Autocell: Entry of a new substrate into the autocell Real cell: Mutations in the genome

Anaphase consists of anaphase A (sister chromatids move towards either spindle pole) and anaphase B (both poles move apart). a) What is one driving force for anaphase A? b) What is one driving force for anaphase B?

a) What is one driving force for anaphase A? Depolymerization of *kinetochore* microtubules from their plus-ends OR Depolymerization of *kinetochore* microtubules from their minus-ends (at spindle poles) by kinesin-13 b) What is one driving force for anaphase B? Kinesin-5 motors sliding apart *interpolar* microtubules OR Cytoplasmic dynein attached to the plasma membrane pulling on *astral* microtubules

. In muscle cells: a) What is one function of the desmin intermediate filaments? b) What is the mechanism that causes muscle contraction after release of Ca2+ from the sarcoplasmic reticulum?

a) What is one function of the desmin intermediate filaments? Align sarcomeres between adjacent myofibrils b) What is the mechanism that causes muscle contraction after release of Ca2+ from the sarcoplasmic reticulum? Ca++ binds to troponin-C Then troponin-C moves tropomyosin and exposes the myosin-II binding site on actin, causing muscle contraction

Briefly provide a structural reason for the extracellular matrix protein properties stated below a) Why is collagen strong and inextensible? b) Why do proteoglycans attract water to form hydrated gels?

a) Why is collagen strong and inextensible? Due to its triple-helix OR cross-linked structure b) Why do proteoglycans attract water to form hydrated gels? Due to their (charged or polyanionic) polysaccharide chains

Histone deacetylases (HDACs)

also dissociates --> histone acetylated, chromatin opens, RNA polymerase complex can move through.

an experiment using Chlamydomonas. b) What does this experiment tell us about how flagella grow? c) What is the name of the microtubule organizing center (MTOC) for flagella? d) What is the function of intraflagellar transport (IFT) in these cells?

b) What does this experiment tell us about how flagella grow? That flagella grow from their tips (or plus-ends) [and not from the ends facing the basal body] c) What is the name of the microtubule organizing center (MTOC) for flagella? ___basal body (not axoneme!) d) What is the function of intraflagellar transport (IFT) in these cells? Transport flagella building blocks [i.e., tubulin, dynein, radial spoke parts, etc] to the growing flagella tip and recycle stuff back into the cell [usually to be degraded].

Pointed and barbed faces of G-actin

barbed face associates with cell membrane, pointed points in towards cell interior.

Thrombin

blood plasma proteolytic enzyme activated by injury (prothrombinthrombin) enzyme that converts fibrinogen to fibrin during coagulation

Arp2/3 complex mediates ___ actin filament nucleation

branched (Lamellipodia)

prometaphase

breakdown of the nuclear envelope phosphorylation of lamination's by cdk once envelope break down, the Mts now have access to bind to chromosomes -- chromsome start to begin to line up

Contact inhibition

cadherins important for cell-cell interactions --> generate intracellular signals that suppress cell division and migration. Problems --> malignancy

Metaphase: Capture of microtubules on one face of kinetochore is followed by

capture on other face. Microtubules shorten and lengthen until the "push" vs. "pull" becomes equivalent, at which time they are localized along the middle("congress"). Interpolar microtubules have motors working against each other, maintain overall size of spindle

The tail domain of motor proteins bind to

cargo.

It's a cell CYCLE, so you can block it anywhere and stop it. This idea used for mutant screen. CDC (cell division cycle) mutants:

cell cycle is a simple dependent pathway

Q: How do actin and MTs mediate cell motility, respectively? Examples

cell motility with actin (lamellipodia, actin polymerization, actin crosslinking) MTS - flaggelium, cilia

Experiment on cell sorting: like-like cells associate;

cells with more cadhedrin associate faster than cells with less cadhedrin.

Primary cilium

centriole serves as basal body. MT doublets but no outer/inner dynein arms or central pair (9+0), so immotile. On most differentiated cells. Often serve as sensory organelles; olfactory and rod/cone receptors are specialized examples. Defects in IFT --> can't construct cilia, including primary cilia --> diseases e.g. retinitis pigmentosum, polycystic kidney disease

Centrosomes (MTOC) - essential?

centrosome is absolutely dispensable in mitosis Note that centrosomes are not absolutely necessary for mitosis- i) Plants completely lack centrosomes ii) Functional spindles can be formed in vitro in the absence of centrosomes iii) Mitosis takes place in cells even after surgically removing centrosomes using lasers --> Centrosome are thought to make mitosis more efficient (faster, more accurate)

General features of DNA damage check point systems Reminiscent of signaling pathways

checkpoints are signaling pathways: designed to sense a problem, sensed by a sensor, will pass message foward to tranducers which will engage effectos to fix the problem stress --> sensors --> transducers --> effectors --> responses

G1 checkpoint for DNA damage (checkpoint #2) p53 and mdm2

checks for DNA damage before it is replicated DNA damage --> kinase (ATM) phosphorylates p53 --> prevents destruction of p53 by Mdm2. - p53 job is to arrest cell in G1 and cell death (apoptosis) *In a healthy cell, Mdm2 E3 ligase keeps p53 levels low* (high p53 levels are toxic) p53 is a tumor suppressor. p53 mutant/deleted in about half human cancers!

Study these adhesion proteins with sponges-What experiment do we use? What evolutionary transition did we discuss in terms of sponges?How does this connect to our current discussion?

choanoflagens, adhesion proteins: helped with multicellularity *Existence first demonstrated with unicellular flagellate cells (like Chlamydomonas) --> tip growth of regenerating flagella.*

Prophase

chromosome condensation begins duplicated centrioles begin to separate microtubules half life decreases and asters form cell begins to round up 1) increased MTOC activity in centrosomes --> more MT 2) dynamic instability greatly *increased* --> short Mts; "searching" for chromosomes

Tight junction proteins

claudin and occludin

Cell Division Cycle

cyclic process •A chain of events that lead to cell division •Problems with cell cycle frequently lead to cancer •Cell division = cell reproduction for unicellular organisms. •In multicellular orgs, cell division creates more cells and/or replaces dead or injured cells.

Summary of cdk-cyclin pairs

domino type effect by cdk-cylin pairs 1. Early G1: Cdk still low, allows chromosomes to decondense, nuclear envelope reassembly. 2. G1/S transition: D cyclins allow a pulse of Cdk4/6 activity --> pass restriction point, switch to Cdk2-cyclin E to allow for DNA synthesis 3. Mid-S --> G2: Cdk2-cyclinE goes, replaced by mitotic (first Cdk2-A, then Cdk1-B) 4. Early mitosis: active mitotic Cdks (Cdk2-cyclinA & Cdk1-cyclinB). At prometaphase, APC/C targets cyclin A --> Cdk2 activity low 5. Late mitosis: At end of metaphase, cyclin B targeted by APC/C. Cdk1 activity now low --> enter anaphase Cdks always present, only active when correct cycle is present

Compared to kinesin, dynein is

dynein is a large, floppy motor dimer--> can switch protofilaments tracks (also, can take backward steps). Gene family also—cytoplasmic dynein, axonemal dynein and IFT dynein

Elastic fibers:

elastic, also made by fibroblasts. Prominent in skin, artery walls, and lung. Important for blood circulation. One of the major proteins that make up elastic fibers in elastin which is highly cross-linked (don't need to know cross-linking details). When stretched--> low entropy ordered; when relaxed --> higher entropy disordered. Elastin only made by fibroblasts in embryos and young people; adults have to do with what they have. Skin wrinkling with aging due to loss. Marfan syndrome: crummy elastic fibers --> aortic aneurysms and general floppiness of artery.

Contractile bundles of actin filaments (actin + myosin)

emergent property; contractile ring

Intermediate filament expression cell-type-specific:

epithelia keratin, muscles desmin. But this can change during differentiation: skin story K14 in basal cell layer --> K10 in spinous layer. Inherited skin diseases, blistering. Different keratin defects give different pathologies owing to different layers of skin express different keratins Hyperkeratosis: excess scaling of skin.

Neurofilaments important in axons

expand diameter, increase rate of action potentials.

In vitro motility assay

filaments move pointed-end leading when myosin-II is tethered.

Major way to figure out mechanism of cell cycle:

fission yeast, cdc mutants; In mutant --> Can cycle to that point and then blocked (screwed!). Examples: wee and cdc25 mutants.

Locomotion by pseudopod extension ( ameboid movement):

growth cones via pseudopods and filopodia, keratocyte via pseudopod. We've seen lots of this already with white blood cells, phagocytosis, etc. Actin filament growth associated with plasma membrane pushes it forward = leading edge. Myosin-based contraction in rear that releases the "tail" end of the cell.

Kinesin structure:

head, neck (lever arm), stalk (coiled-coil), C-term with light chains bind cargo. Heads bind to beta-tubulin --> need to take large steps compared to myosin II.

F-actin:

helical structure. Pointed and barbed faces of G-actin are found at the pointed and barbed ends of a filament (F-actin) i.e. filament has polarity, as seen by myosin head fragment decoration

positively charged, hydrophilic "basic amino acid" 3 HRK

histidine (his) arginine (arg) lysine (lys)

Aortic aneurysms are large "bulges" in the walls of the aorta, the major artery carrying blood from the heart to body cells. When an aortic aneurysm ruptures, it has a nearly 98% chance of fatality. The rupturing occurs because of both a weakening of the aortic wall and a decrease in its ability to stretch. Each of these situations is due to the degradation of ECM proteins. For each situation, suggest an ECM protein whose loss contributes to the rupture, and explain your answer. i. Weakening of aortic wall ii. Decrease in stretching ability

i. Weakening of aortic wall: Degradation of some kind of collagen, which is required to strengthen the aortic wall (because collagen is inextensible). ii. Decrease in stretching ability: Degradation of elastic fibers (or their components), which provide elasticity.

Inactivation and reactivation of Cdk1/cyclin B

in G2, after CAK activation, specific residues on Cdk get phosphorylated by Wee1 --> blocks the kinase activity; later a phosphatase Cdc25 takes them off and Cdk active again. We'll see the importance of this step later.

Fibrinogen

is a dimeric blood plasma protein; has 2 RGD domains--> link platelets into aggregates A blood protein essential to blood clotting. The conversion of fibrinogen to its active form (fibrin) is among the final steps in clot formation, and is triggered by thrombin.

Vertebrates: 5 important families:

keratins, desmin, vimentin, neurofilaments, and lamins (from nucleus part of course). Expressed in different cell types. Can be important in diagnosing cell-type origin of tumors

γ- tubulin ring complex (γTuRC): microtubule nucleator

key regulators, number of microtubule nucleated in cells Yellow subunits --> γtubulin. γTuRC acts as foundation for the assembly of a new microtubule Note the washer-ring like structure --> matches 13 protofilament number.

metaphase

kinetochores microtubules astral MTs interpolar mts chromosome congregation (line up in the center of spindle to form metaphase plate) last checkpoint: Spindle assembly checkpoint (SAC), make sure its lined up correctly

Relationship of arrowhead configuration to myosin stroke pattern (power stroke)

lever arm; power stroke that causes motor to move Basis for arrowhead configuration of myosin on actin filaments: myosin heads in their M or M-ADP configuration bind tightly to actin filament

Kinesin family

lots of them. Usually homodimers.

Myosin family tree

lots of them; they all have similar head domains (whether monomer or dimer), but vary in tail domains.

F-actin capping proteins

maintain filament length Bind either barbed or pointed ends and block addition and subtraction when bind. Capping protein: barbed end Tropomodulin: pointed end. We'll see this in skeletal muscle. in muscle Way more G-actin than F-actin cells. Why? --> Because of monomer-binding (cofilin) and capping proteins --> System ready to rapidly elongate any free barbed ends or generate new filaments

Anaphase

metaphase has to occurs correctly to bypass checkpoint to get to anaphase sister chromatids separate - anaphase A: chromatids approach pole (APC/C active) - (poles not moving, chromatid separating) - anaphase B: spindle *poles* migrate apart

Centrosome with centrioles and pericentriolar material

microtubule based structure PCM: gamma tubular ring complex

Cytoskeleton components

microtubules (hollow tubelike- pipe), actin/microfilaments (thinnest), intermediate filaments

Phosphorylation of Rb is triggered by

mitogenic signals that we've thought about before, coming in from RTKs (MAP kinase cascade), steroids, 7-helix receptors (PKA), and from integrins --> results in synthesis of cyclin D --> activates Cdk4/6 --> enter nucleus and phosphorylates Rb, allowing it to dissociate from E2F

Dynamic instability in vivo:

most interphase microtubules a half-life of ~10 min. Much more labile in mitotic cells. Some MT in interphase cells last longer due to post-translational modifications and plus-end cappers (later).

Cytokinesis, early and late

needed to avoid polyploid cells( multi nucleated) early: - new membrane inserted, - *acto- myosin* (key players) contractile ring forms (pinching force) - midbody begins late : - chromosome decondenses - nuclear substructure reform - interphase microtubule array reassembles midbody - vesicle fusion drives separation (abscission) of the two cells

Dynamic instability of microtubules in *living cell*

not only dynamic instability in vitro

Platelets carry 2 kinds of integrins

one binds to the RGD on fibrinogen; the other binds to Type IV collagen in basal lamina

G2 checkpoint (checkpoint #3) for DNA damage, completion of DNA (You are not responsible for this whole pathway!)

overall, its the same as the G1 checkpoint; thats all you need to know fr 1) Detect problem 2) Activate kinases, starting with ATM again 3) Keep cell from entering mitosis until repair, or signal apoptosis through p53 When this checkpoint fails, daughter cells carry mutations!!

G1 DNA checkpoint has 3 key players:

p53 (transcription factor and our second example of a tumor suppressor), Mdm2 (an E3), and ATM, a kinase. In healthy cell, p53 held to low levels by Mdm2 which ubiquitinates it, in a complex cool pathway that involves going in and out of the nucleus. When there's DNA damage, p53 is phosphorylated by a bunch of kinases, the one to remember being ATM. When p53 phosphorylated--> Mdm2 can't bind to it and p53 levels rise, allowing it to activate expression of a bunch of genes that lead to either cell-cycle arrest while DNA damage repair attempted, apoptosis if that fails.

Actin filament: F-actin

polymer of g actin g actin pile on end to end but with a slight skew that gives rise to a helical pitch

Kinetochore structure

protein based structure, multiple layers Kinetochore assembles in association with centromeric DNA how does it capture Mts?-- Corona of motors (e.g. CENP-E, a *kinesin*) on kinetochores before microtubules are attached Many other motors are involved in mitosis!!!

Microtubule associated proteins, MAPs (> 100)

regulates the creation, length, and stability/lifetime of microtubules MAPs regulate MT behavior by binding to the MT tips or to the sidewall

telophase

resetting things back to a interphase state Cyclins A and B are by now destroyed due to APC/C ubiquination --> phosphatases remove phosphates --> lamins reassemble and microtubules return to interphase behavior. cleavage plane specified

Phenotypes of cell-cycle mutants in *fission* yeast

rod shaped eukaryote cell - wee mutant: accelerate g2 --> M; divide before they reach final size that is appropiate, making cell size small - cdc25 mutant : inhibits g2 --> M; continuous growth before division, making cell size v long

During RNAi, how does the RISC complex selectively binding to a target mRNA?

siRNA in the RISC complex base pairs (or hybridizes) with the target mRNA

APC/C and chromatid separation

sister chromatids are associated along their entire length (not just at kinetochore) by *cohesion complex* -- have to break cohesion complex to separate -- protease is called *separase* to enable separation of sister chromatids -- separase is inhibited by *securin*, inhibits this protease at all other stages APC will polyubiquinate securin and cyclin B --- destroys securin allowing separase to work APC/C causes securin to be degraded --> separases active --> breaks cohesion ring

results from Under-express Wee1 --> overexpress Wee

small cells; overexpress Wee1--> extra large cells.

Kinesin-1 structure

smallest and fast motor proteins ( like the ferrari of motor proteins dimer head, coiled coil, tail neck (lever arm part)- power stroke Light chains help connect kinesins to cargo 2 foot walk

Proteoglycans

space-fillers, water retainers - These proteins have lots of polysaccharides attached to them. - Glycosaminoglycans (GAGs) are one type of polysaccharide that are attached to proteoglycans. - Unlike collagen and elastic fibers, made by all cell types in vertebrates. - Major constituents of cartilage, loose connective tissue, and basal lamina. Names of GAGs and proteoglycans and not worth trying to remember at this stage in your training unless this is your thing. GAGs have lots of negatively charged sugars and sulfates attached making them polyanions, therefore extended by repulsion and attract large amounts of water and take up lots of volume. Lubricants in joints.

+TIP proteins (e.g. EB1)

specifically bind to the plus-end of growing microtubules, helps stablilize growing ends (typically they promote microtubule growth). Also mediate association of MTs to membranes (eg., plasma membrane, ER). These proteins "find" the growing MT plus-end by specifically recognizing and binding to the GTP cap structure.

*interpolar* motors work against each other, maintaining overall length of the spindle

spindle shortening forces: 1) kinesin-13 (spindle poles) --> depolymerization MTs at poles (generated at both poles, motor proteins at the center, depolymerize at the minus end, work to make shorter) 2)kinesin- 14 (interpolar) --> inward force (attach to interpolar Mts at the overlapping regions (interdigitated part))- squish them together outward acting forces vvv Spindle lengthening forces: 1) kinesin-5 (interpolar) --> outward force (plus end directed motors) 2) cytoplasmic dynein (PM) --> pull astral MTs

Integrins Focal contacts

stick cells to ECM. Once integrins bind to ECM, signal transduction --> a bunch of tyrosine kinases activated --> get focal contacts. This often tells cell to alter gene expression

Assembly of intermediate filaments:

tetramer, no polarity - antiparallel fashion emergent property: tensility

Non-motile MAPs like tau can significantly affect

the motility of motor proteins like kinesin. Dynein motility less affected by tau and shown in movie.

Snake venoms often contain small proteins called disintegrins that contain RGD motifs

these compete with fibrinogen for binding to platelet integrins inhibit blood clotting disintegrins (RGD) dont form dimers, only bind single proteins (monomeric) and prevent blood platelets from crosslinking with other platelets and clotting (like fribrin)

Types of Junctions

tight junctions, desmosomes, gap junctions

Formin mediates ___ actin filament nucleation and elongation

unbranched (stress fibers)

Cdk so-named because only active if associated

with cyclin proteins (cyclin-dependent), 16 cyclins in humans, some in G1, some in G2 etc. I'd like you to remember that - *cyclin B goes with Cdk1*, - *cyclin A and cyclin E go with Cdk2*, - *and cyclin D goes with Cdk 4/6*.

Tubulin structure

α-and β-tubulin 40% identical. Form very stable heterodimer β- tubulin's GTP exchangeable, α-tubulin's GTP non-exchangeable Plus (beta) and minus (alpha) ends - polar filaments (has directionality) - both subunits bind GTP, only beta can be hydrolyzed and exchangeable

Much of the pericentriolar material is comprised of

γ-tubulin ring complexes γ-tubulins assemble with other proteins to form little rings that nucleate microtubule assembly. Hence the centrosome in general is said to be a microtubule organizing center (MTOC), with the ring complexes being the active ingredients in this activity, associating with MT minus ends. The centrosomes of mitotic cells acquire far more γ-tubulin, allowing them to initiate many more microtubules.

3 Cdks and 4 cyclins to know Cdk1 Cdk2 Cdk4/6

• Cdk1 goes with (is regulated by) cyclin B: mitotic • Cdk2 goes with (is regulated by) cyclin A: mitotic • Cdk2 also goes with (is regulated by) cyclin E: G1/S • Cdk4/6 goes with (is regulated by) cyclin D: mainly G1

Ig-CAMs (ICAMs)

• Extracellular Ig-like domains •Large gene family • Single TM segments • Varied cytoplasmic domains

Tools for investigating cytoskeleton binding proteins Actin-binding drugs

•*Cytochalasin* Binds barbed ends and leads to formation of ADP-actin pool --> *inhibits polymerization* •*Latrunculin* Binds monomers (G-actin) and inhibits polymerization -->Net effect of *actin depolymerization* •*Phalloidin* Binds and stabilizes F-actin filaments, can be labeled with a fluorescent dye --> *stain actin in cells*

Kinetics of F-actin elongation

•Actin polymerization rates different at the two ends •To get polymerization, need to exceed critical concentration (K). ATP-actin much more effective(actin saturated with ATP in cells) •Critical concentration for T form (ATP) is less at barbed than pointed endadds faster at barbed end - barbed end has much more growth than pointed end

Six general principles of adhesion

•Cells produce *cell-type-specific* adhesion molecules during differentiation (this defines what they can bind to; proper cell types need to associate at the right time and space during development!) •*Binding preferences*: cadherins prefer themselves (homophilic); selectins bind mucins; Ig-CAMS bind other proteins (heterotypic). Integrins bind lots of things •*Modulate adhesion* by controlling surface density, state of aggregation, and state of activation •*Binding usually weak*, compensated by many molecules participating (think velcro) •Many adhesion proteins *interact with cytoskeleton* (via adapter proteins) for mechanical support; interaction with cytoskeleton provides mechanical continuity from cell to cell (eg., in muscles and epithelia). •*Adhesion can generate signal transduction* --> respond to physical interactions with ECM or neighboring cells

Platelet activation, aggregation Resting state

•Collagen receptor active, but no ligand exposed --> no sticking to BL •Fibrinogen integrin inactive -->no platelet pileup •Low concentration of soluble activators (ADP and thrombin) --> can't activate fibrinogen integrin

Platelet activation, aggregation Activation and aggregation

•Damage to endothelium--> BL exposed --> platelets tether to BL by binding to collagen (and von Willebrand factor by gp1b)- platelets are activated --> secretes ADP and thrombin --> they bind to their receptor on platelets so fibrinogen can bind, fibrinogen is also activated by thrombin and binds which links platelets •Binding to BL --> fibrinogen integrin activated from ADP secreted from platelets --> ADP binds to its receptor --> activates fibrinogen integrin •Damage to endothelium --> thrombin --> thrombin binds to its receptor --> activates fibrinogen integrin - activated fibrinogen integrin bind to the rgd motif on platelets causing platelets to adhere to each other. •Platelet aggregation --> further stimulates response to ADP and thrombin

Katanin: a microtubule severing protein

•Kataninis a AAA ATPase. •Assembles on sides of microtubules as a *hexamer* -->ATP hydrolysis drives microtubule severing (likely by removing a tubulin subunit) each monomer is a ATPase breaking microtubules, ATP dependent rxn

Dynein structure

•MT-binding domain located at the tip of the coiled-coil stalk •The linker domain is the mechanical element that is responsible for the power stroke Dynactin complex links dynein to vesicles

Actin nucleotide exchange is regulated by actin binding proteins

•Profilin promotes all forms of nucleotide exchange; ADP -->ATP and ATP -->ADP •There is way more ATP than ADP in the cytoplasm & actin has a higher affinity for ATP --> profilin maintains actin monomers in the AT P-bound state and ready for polymerization. •Pro-filament = profilin •Cofilin inhibits nucleotide exchange - Binds to ADP-actin and keeps in ADP-bound state and unavailable for polymerization.

GTP cap hypothesis

•Tubulin polymerization: β-tubulin GTP GDP•GDP polymer under stress (due to "curved" tubulin) •GTP cap at plus-end stabilizes it. •GTP-cap loss --> catastrophe -- GDP state: peels back bc curve conformation, strains lattice --> work to pull apart the lattice rescue --> elongation


Set pelajaran terkait

Level 1 - Chap 5: Leasehold Estates

View Set

Medical Terminology: Chapter 19 Questions, med term chapter 1-19

View Set

Communications Midterm MC CHAPTER 9

View Set

Intro to Programming - Chapter 7

View Set